LSAT Logic Lab
LSAT Logic Lab
LOGIC LAB
Chapter 1 - Introduction
Overview of the LSAT
Overview of Logic Games
Congratulations on your decision to take the next step in pursuing a career in the legal
profession. This text is designed to accompany our minicourse in the analytical
reasoning, or logic games, section of the Law School Admission Test (LSAT). As you
probably know, satisfactory performance on this test is necessary for admission to
law school programs approved by the American Bar Association. To do well on the
overall exam requires that you make a reasonable score on the logic games part. While
the logic games are only one of several sections in the overall exam, many people who
have taken the LSAT regard it as the most challenging section.
Our immediate purpose is to give you practical instruction and experience in applying
strategies for solving logic games problems, but our ultimate goal is to help you
develop confidence in knowing what to expect from the exam and in applying your
newly acquired knowledge so that you can do your very best.
Neither of us are attorneys, however our minicourse has been developed with the
assistance of two attorneys with decades of combined professional experience. We
are mathematicians with substantial backgrounds in teaching all aspects of applied
logic. We sincerely believe that this gives us a unique advantage in communicating
quickly and efficiently the basic knowledge that is required for maximizing
performance on the analytical reasoning component of the LSAT. We have taken many
standardized tests in our careers, so we can appreciate any apprehension you may
have about these tests, in general, and the LSAT, in particular. Our job is to help you
rise above it.
There are several LSAT review courses available from national providers featuring
various blends of self-study, distance learning, and traditional classroom instruction.
Although they tend to be complete and informative, they also tend to be expensive,
time intensive, and in some cases, rather inconvenient or impersonal. We have tried
to identify and further develop some of the strengths of these courses so that they
can be presented at a reasonable pace in a teaching environment featuring
personalized support, immediate feedback and reinforcement from instructors, and
collaborative engagement and learning with peers. We have taught mathematics with
this approach for many years with a fair amount of success.
We hope this little book and the minicourse are just what you need to succeed in
taking that next step.
INTRODUCTION
The LSAT (Law School Admission Test) was originally developed by representatives
from several elite law schools (Harvard, Yale, and Columbia included, of course) and
first administered in 1948. There was a perception that the admissions exams in use
at the time were not sufficiently correlated with first year grade performance. A
committee was formed to draft an exam that would correct this, although some
participating law schools were openly skeptical about the admission of applicants on
the basis of “aptitude”. In retrospect, this seems quaint.
Preparation and administration of the test is currently managed by the LSAC (Law
School Admission Council), and there are four test dates every year: February, June,
September/October, and December. Pending sufficient registration, the FGCU LSAT
minicourse will be offered on Saturdays the month prior to each test date. Currently,
about 150,000 examinees take the test in a given year, although it has peaked at over
170,000 in the past. Most examinees are from the United States, although the LSAT
is being used increasingly in Canada and other nations in the Anglosphere.
The LSAT has evolved quite a bit since its early years. The “modern era” of the LSAT
started with the June 1991 test, which was labeled “Prep Test 1". The basic form of
the exam has remained relatively stable from that point on, and all subsequent tests
have been numbered consecutively, so that the February 2010 test became “Prep Test
60". Unlike, for example, the testing bodies that administer exams for the
engineering profession, the LSAC is quite cooperative about making prior exams
available to the public, although some particular exams are no longer accessible. In
fact, the LSAC recommends that examinees study the prior exams and it even makes
available a practice exam at the LSAC website. The sample problems in this book and
the worksheets distributed in our minicourse use these prior exams freely as a
resource, as do the big test prep firms such as Kaplan and Power Score.
SCORES
The scoring system for the LSAT goes from 120 to 180. The LSAC takes raw scores
(questions answered correctly) and adjusts them with a conversion formula to account
for variability in the difficulty level of the exams from year to year. This ensures
some degree of comparability of scores over time. Typically, the 50th percentile score
is about 151, the 90th percentile about 163, and 99th percentile about 172. The “sooper
genius” percentile (99.9th) requires a score of 178.
A study at the University of North Texas has focused on the distribution of average
LSAT scores by academic major. The findings are intriguing: criminal justice majors
average 146.0, pre-law majors 148.3, liberal arts majors 152.4, English majors 155.2,
and (sorry) mathematics majors 160.0.
While the LSAT is currently required for admission to law schools accredited by the
American Bar Association, there has been some discussion regarding making them
optional. This parallels the current debate over making the SAT and ACT optional in
order to promote diversity of the student body. Georgetown and the University of
Michigan have waived LSAT scores for students with sufficiently high GPAs.
As far as the original purpose of the LSAT - to predict first year law school grades -
the LSAC has studied the issue and reports that there is about a 40% correlation
between LSAT score and first year GPA. The claim is that this is a better predictor
than baccalaureate GPA. Evidently a combination of undergraduate GPA and LSAT
score, called an admission index, is even better at predicting first year grades. There
is some small variability in the correlation depending on the particular law school
attended.
There is a rule concerning how many times you can take the LSAT - no more than
three times in two years unless specifically granted an exemption. Also, unlike the
SAT, the LSAC reports all the scores that you have gotten in the preceding five
years to the schools you have chosen as score recipients. You can’t “hide” a poor
showing, although if you are absolutely sure you don’t want the results recorded for
a test you have just completed, there is a provision on the answer sheet to basically
pretend it didn’t happen. That particular score will not be part of any score report,
and, in fact, LSAC will not even report it to you. This is the nuclear option, so you may
want to consider this very carefully. Law schools have different policies regarding
which score or scores to consider if they receive multiple scores.
FORM OF THE TEST
The LSAT has six parts: two sections of logical reasoning, one section of analytical
reasoning, one section of reading comprehension, one section that can be any of the
three preceding types, but which is not scored, and one section consisting of a writing
sample, which always comes last. The order of the other sections may be shuffled to
improve test security. The unscored section is considered experimental in the sense
that responses to questions on it may guide the preparation of future test questions.
Examinees are not told which section is the unscored one.
It may seem that logical reasoning and analytical reasoning should be one and the
same, but that is definitely not the case. In the logical reasoning part of the exam ,
a short argument is stated and then the examinee is asked to identify assumptions,
errors, and the effect on the strength of the argument if certain premises are
added or removed. Common sense and a little practice with the format is excellent
preparation. Our minicourse covers logical reasoning in the afternoon session.
In the analytical reasoning part of the exam, various puzzles known as “logic games”
are presented. They are the subject of this book. Logic games are a bit like crossword
puzzles, Sudoku™, or the word game Jumble™ in that they have partial information
given in the form of clues which fit a predetermined game structure. The challenge
is to fill in the blanks, so to speak, and answer various questions about that structure.
The difficulty level of the games is determined by the exact nature and complexity
of the game format, the quantity and helpfulness of the clues, and the depth of the
questions that must be resolved. We will analyze these issues in complete detail
shortly.
The reading comprehension section, which is also addressed by our minicourse in the
afternoon session, consists of four short (about 500 words max) articles on various
academic subjects that would be reasonably familiar, for example, to a liberal arts or
pre-law major. The questions center on determining the main theme of the article,
correctly identifying facts that are presented, and making valid conclusions from the
information given in the text. Obviously, the ability to read quickly and with decent
comprehension is the key to handling these tasks in the time frame allowed. Skimming
for critical information is a skill that can be learned.
The writing sample section asks you to take a pro or con position on a fairly benign
subject and defend your choice as eloquently as you can manage after the grueling
multiple choice part of the exam is over. The LSAC used to administer the writing
sample part at the beginning of the overall exam. This section of the LSAT is actually
not evaluated by the LSAC as part of your score report. An electronic image of your
response is sent along with your score report to the various law schools you have
designated. Those institutions are free to use or ignore your essay, but you should
write something, since the LSAC may not report your scores if you don’t attempt the
writing sample. Schools that require other written material, such as a “statement of
intent” or similar personalized essay expressing your interest in a law career, may
typically be the ones to ignore your LSAT essay.
The current cost to take the LSAT is about $140. The testing center will want to take
a thumbprint for positive identification, in addition to seeing some form of
government issued photo identification, such as a driver’s license, passport, or military
ID.
The cycle for the four annual tests begins with the June exam, followed by the fall
exam which may occur in September or October, then the December exam, and finally
the February exam. The fall exam is the most popular with examinees intending to
enter law school the following fall. December and February exam dates provide an
opportunity to shore up scores that may need to be improved to allow a realistic
chance at a top school.
The test itself consists of five 35 minute multiple choice sections totaling just about
100 questions plus a final 30 minute writing sample section. There is a short break
given in the middle of the exam. There is no penalty for wrong guesses, so you should
save a little time to fill in the answer sheet no matter what with your best guesses
after you have made all the responses in which you have some confidence. Poker is as
much about money management as card management, and tests of this nature are as
much about time management as anything else. Using well the rather short amount of
time allotted for the multiple choice sections is one of the key principles we will
stress.
INTRODUCTION
The analytical reasoning, or logic games (LG) section of the LSAT consists of four
self-contained games with typically five to eight multiple choice questions about each
game. The multiple choice format is five possible responses per question. The total
number of questions is usually about 24. You have 35 minutes for these questions, so
that is an average of about a minute and a half each to respond to everything. This
includes the amount of time you need to read and understand the game scenario. We
mention this not to alarm you, but to reinforce the point that time is very much of the
essence. If you are not by nature a quick thinker, and most of us are not, you may
take some comfort in considering that if you slow down to make sure that what you
do is correct, even if you only answer half of the questions right on the test overall,
your score would still be about 150.
The games themselves are logic puzzles of very specific configurations that the test
authors use year after year. The common thread is that a puzzle presents a little
story that defines various people or things which appear and the nature of the
relationships among them. The little story is called a “scenario” and the people or
things are called “variables”. The variables are given to you in the scenario so there
is no confusion, but in some cases you may have to decide how to symbolize them with
single letters in order to work with them efficiently. Next after the scenario is a
listing of various conditions, or “rules”, which govern the allowed or possible
relationships among the variables. These are generally fixed for the entire game and
are called “global” rules. Finally, there is a list of five to seven (rarely eight) multiple
choice questions which ask you to determine other facts or relationships that are
consistent with the given rules. Typically the first few questions depend only on the
global rules, but later questions may also introduce a new “local” rule just for that
question, or suppress one of the global rules just for that question.
While one question for a given game may seem completely independent of another
question, it is frequently the case that having the answer to one vastly simplifies the
search for an answer to the other. Keep this in the back of your mind. The test
authors can be a little diabolical and they know, of course, that you are under severe
time pressure. Often they plant a question toward the beginning that is not
intrinsically difficult, but takes a little time to chase down and reject a number of
incorrect answers. Learning how to recognize a “time-waster” will be something we
want to practice.
TYPES OF GAMES
Among the game configurations that have appeared during the last twenty years of
the LSAT, four main ones and their variants account for substantially all of the
problems used in the analytical reasoning section. These are: (1) linear games, (2)
grouping games, (3) multilevel linear games, and (4) combination linear and grouping
games. There are a few formats that have appeared very infrequently, such as games
based on sequencing, and we will mention them briefly but spend the bulk of our time
on the main four above.
The best way to understand what these game formats are is to look at some examples:
1) Linear Games
Linear games require you to arrange variables in a single sequence subject to all the
constraints given.
RULES:
1) J is not advertised during a given week unless H is advertised during the immediately preceding week.
2) The product advertised twice is advertised during week 4, but is not advertised during week 3.
3) G is not advertised during a given week unless either J or else O is advertised that week.
4) K is advertised during one of the first two weeks.
5) O is one of the products advertised during week 3.
The linear nature of this game is apparent from the string of four successive weeks.
The problem consists in figuring out how to distribute the advertised products over
the four weeks so that the constraints are satisfied. We are “lining up” four pairs of
products in a sequence. Whenever there is an explicit or implied time sequence in a
scenario, you can be pretty sure you are dealing with a linear game.
Which of the following is a pair of products that CANNOT be advertised during the same week as each
other?
a) H and K
b) H and M
c) J and O
d) K and L
e) L and M
To answer this question, you would have to try to configure the product pairs over
the four weeks obeying all of the constraints. You may find in trying to do this that
one of the five pairs above cannot appear without violating a constraint. That would
be your answer. We will solve this game in detail later using our strategies for linear
games.
2) Grouping Games
Grouping games (pure grouping games, not the linear/grouping combo games discussed
subsequently) present a set of variables and ask you to assign them to subsets
according to rules that link individual variables. There is no concern for any type of
sequential order as with linear or multilevel linear games. Matching games fall into
this category.
SCENARIO: A university library budget committee must reduce exactly five of eight areas of
expenditure - G, L, M, N, P, R, S, and W - in accordance with the following conditions:
RULES:
1) If both G and S are reduced, W is also reduced.
2) If N is reduced, neither R nor S is reduced.
3) If P is reduced, L is not reduced.
4) Of the three areas, L, M, and R, exactly two are reduced.
The task presented by this game is to make an assignment of the eight variables to
two groups: those reduced and those not reduced. Many assignments would be
disallowed by the constraints. Although it may not seem like it, there are enough
constraints in this problem to whittle down the possible assignments so that we could
identify a definite subset of five areas that would be reduced (and by implication
three that would not).
If P is reduced, which one of the following is a pair of areas of expenditure both of which must be
reduced?
a) G and M
b) M and R
c) N and R
d) R and S
e) S and W
This question falls into the category of “game changer”. In fact, five of the seven
questions posed for this game are of this type, where new information in the form of
a hypothetical (“If P is reduced...”) changes the basic rules of the game. This is a very
common occurrence in the logic games.
Multilevel linear games (some test prep authors call them stacked linear games)
require you to arrange sets of variables in two or more sequences with all constraints
satisfied. There is some interdependence among the variables in different sets
established by the language of the constraints, so the sequences need to be
coordinated.
SCENARIO : Eight physics students - four majors: Frank, Gwen, Henry, and Joan; and four non-majors:
Victor, Wanda, Xavier, and Yvette - are being assigned to four laboratory benches, numbered 1 through
4. Each student is assigned to exactly one bench, and exactly two students are assigned to each bench.
Assignments of students to benches must conform to the following conditions:
RULES:
1) Exactly one major is assigned to each bench.
2) Frank and Joan are assigned to consecutively numbered benches, with Frank assigned to the lower
numbered bench.
3) Frank is assigned to the same bench as Victor.
4) Gwen is not assigned to the same bench as Wanda.
Although a time sequence is not evident in this problem, it is clear that the sequence
of numbered benches will do. The problem author gives us a big hint here. Note that
there are two groups of variables that are separated by a condition...being a major or
non-major. And note how thoughtful the test writer was to invent names that would
allow us to assign variable letters F, G, H, and J versus V, W, X, and Y. We recognize
this problem as a multilevel linear game, since we are tasked with arranging each set
of variables separately with regard to the benches 1 through 4. The two sequences
will not be independent...this is what makes the problem tough...because there are
connections between them that have to be respected. Constraints (3) and (4) do
precisely that by requiring the linking F and V and forbidding the linking of G and W.
If Victor is assigned to bench 2 and Wanda is assigned to bench 4, which of the following must be true?
To address this question you would need to lay out the skeleton of a sequence for
majors and one for non-majors with the definite assignment of V to position 2 and W
to position 4 in the non-major sequence. Then, coordinating the two sequences so that
all of the given constraints are observed, one of the five answers must be unavoidable.
You can be confident that a “which of the following must be true (or false)” type
question will mercifully have only one answer.
Both ideas of linear and grouping games can be combined. In these games, variables
are sorted into groups and then the variables within a group are arranged in a linear
order which is compatible with the constraints of the problem. So the grouping part
of the solution precedes the linear sequencing part of the solution. Although it may
seem that a game with a complicated structure and many constraints would always be
the most difficult, the silver lining is that more structure and rules often combine to
reduce the number of possibilities for the variable configurations. The methods we
will discuss for linear and grouping games carry over to analysis of combo games.
This is a combo game from the February 1994 LSAT:
SCENARIO: A soloist will play six different guitar concertos, exactly one each Sunday for six
consecutive weeks. Two concertos will be selected from among three concertos by Giuliani - H, J, and
K; two from among four concertos by Rodrigo - M, N, O, and P; and two from among three concertos by
Vivaldi - X, Y, and Z. The following conditions apply without exception:
RULES:
1) If N is selected, then J is also selected.
2) If M is selected, then neither J nor O can be selected.
3) If X is selected, then neither Z nor P can be selected.
4) If both J and O are selected, then J is played at some time before O.
5) X cannot be played on the fifth Sunday unless one of Rodrigo’s concertos is played on the first
Sunday.
If the six concertos to be played are J, K, N, O, Y, and Z and if N is to be played on the first Sunday,
then which one of the following concertos cannot be played on the second Sunday?
a) J
b) K
c) O
d) Y
e) Z
This question requires that you have in mind tentative layouts of concerto selections
for the six dates that satisfy the constraints plus the additional condition that N is
played on the first date. You can see that the grouping process...picking the six
concertos to be played out of the ten available...is aided considerably by the extra
rule presented in the question itself. If N must be played, then by constraint (1) so
must J, then by constraint (2) M must not, and so forth. Then as we attempt to
arrange the concertos selected into a sequence, we will discover that one of the
answers, if it is required to be played on the second date, is incompatible with all of
the constraints. We will explore general methods to attack games like this shortly.
5) PATTERN GAMES
Pattern games are a variant of multilevel linear games where the constraints apply to
the variables across the board rather than to one or perhaps several variables only.
For example, you might be told that no variable can appear first in both sequences,
rather than variable A must appear second in at least one sequence.
Here is a pattern game from the October 1997 LSAT:
SCENARIO : Five candidates for mayor - Q, R, S, T, and U - will each speak exactly once at each of
three town hall meetings - meetings 1, 2, and 3. At each meeting, each candidate will speak in one of five
consecutive time slots. No two candidates will speak in the same time slot as each other at any meeting.
The order in which the candidates will speak will meet the following conditions:
RULES:
1) Each candidate must speak either first or second at at least one of the meetings.
2) Any candidate who speaks fifth at any of the meetings must speak first at at least one of the other
meetings.
3) No candidate can speak fourth at more than one of the other meetings.
If the order in which the candidates speak at meeting 1 is R, U, S, T, Q, and the order in which they
speak at meeting 2 is Q, R, U, S, T, which one of the following could be true of meeting 3?
a) Q speaks first
b) R speaks third
c) S speaks first
d) T speaks second
e) U speaks fifth
We will use a modification of our strategy for multilevel linear games to deal with
pattern games.
Occasionally, the LSAT presents circular linear games, sequencing games (distinct
from linear games), or mapping games. We will cover these briefly in the text.
These are just linear games with the beginning and end of the natural order
connected. An example would be a game based on a seating arrangement for persons
around a circular table. Apparently the LSAT has used this game format once in the
last twenty years.
7) SEQUENCING GAMES
This game format has appeared much more frequently than the circular linear game.
Sequencing games resemble ordinary linear games except that the positions of the
variables are relative only to one another and not to some fixed natural order such as
days of the week or fixed rows of seats, for example.
8) MAPPING GAMES
GAME STATEMENTS
The vast majority of logic games that appear on the LSAT are puzzles based on two
general ideas...order and association. Solving such a puzzle amounts to identifying a
typically small collection of objects called variables and then arranging them in a
certain way (order) or grouping them together in a certain pattern (association) so
that all of the arbitrary rules that constrain possible solutions are obeyed. The
introductory patter that defines the variables and describes the basic task of the
game is called the scenario. The scenario and the rules make up the game statement.
The test authors may create games based on simple orderings (linear games),
unordered groupings (grouping games), simultaneous orderings (multilevel or stacked
linear games), and ordered groupings (combination games). Once you gain some
experience in handling problems based on the two main concepts of order and
association, you will find that extending your problem-solving skills to games that are
offshoots or combinations of these concepts will be easier. You will also be able to
immediately classify a game as to its type, and this will save time...always a good
thing...in organizing your approach to solving the game.
SCENARIOS
Logic games are introduced in the form of a scenario, or short paragraph that defines
all the variables and the initial relationships that must be established among them.
Scenario comes from the Italian “scaena” for scene or stage, and it is particularly
apropos in this context. A logic puzzle could be correctly and completely given in
terms of nothing but dry as dust mathematical symbols, but unless you are used to
this sort of thing, this would stimulate no immediately useful imagery in your mind and
would make the crucial details of the game less easy to remember and work with. The
test authors recognize this and try to make the details of the game statement more
memorable by inventing a little story and couching the game in more familiar and
concrete terms. Allowing the game to come alive on stage in your imagination, if only
for the few minutes you need to answer the game questions, is a skill that you will
want to cultivate.
Memory experts will tell you, the more links you can make between new material and
your own experience, the better your retention will be. After all, the faster you can
absorb the big picture, the sooner and more confidently you can move on to answering
the questions. Another benefit of being able to quickly relate to the basic elements
of a logic game is that the questions asked will not be, except in rare (but most
welcome) instances, of the “look up” type. This is a question where a single look back
to the scenario or rules nails down an answer. The majority of logic game questions
require the construction of a solution template, or diagram, which incorporates both
the given conditions of the scenario and the constraints embodied in the rules. Often
there are subtleties in drawing an acceptable template that are apparent only if you
have thoroughly digested the game statement.
VARIABLES
Identifying the variables in a logic game is usually cut-and-dried. Either the test
authors just go ahead and give you symbolic variables...capital letters...or they give
you a string of names that are conveniently chosen so that the initial letters are a
successive string in the alphabet, in which case you simply use those. In doing
problems, we have occasionally seen the case where one letter is skipped over in the
variable list, and unless you catch it, you may jump in and construct a solution template
that has a serious error.
Occasionally you may encounter a game which seems to have two sets of variables and
it is not clear how to proceed. Typically, this happens with a linear game which has a
scenario that presents both objects that are to be arranged in some order, and also
the positions available (in time or space or some other order) for this to be done. You
can resolve these cases easily by focusing on which apparent set of “variables” has a
natural order...like days of the week, rows of seats, and that sort of thing. The
objects with the natural order form the positions in the template into which the
objects that have no natural order are to be arranged. For example, if a doctor was
to see one patient per day among Al, Bob, and Charlie on Monday, Tuesday, and
Wednesday, it is the people who are variables, not the days, which will be naturally
ordered positions in the solution template. Another clue may be found in the rules.
Generally, the variables appear as subjects in the sentences giving the contraints.
RULES
The difficulty of logic games is contained in the scope and subtlety of the rules which
limit your freedom in constructing valid solution templates. It would be easy to seat
five people on five chairs. Less so if we have to worry about seating them so that two
particular people can’t be adjacent and one specific person has to be in the middle.
The number of feasible arrangements plummets as soon as we start imposing extra
conditions. We could easily put so many conditions on the seating problem that there
would be no solution. Obviously the LSAT test designers don’t want to go that far, but
there is a happy...for them...medium, where the solution to such a problem would exist
(there could be several) but be far from obvious.
QUESTIONS
GENERAL OBSERVATIONS
The Logic Games part of the LSAT typically presents twenty-four questions overall
covering four separate games. Occasionally the total number of questions will vary
slightly, but the number of games is always four. Individual games have between five
and eight questions, with eight being very rare. A typical pattern is to have a five
question game, two six question games, and a seven question game. Having the larger
number of questions does not automatically mean that the game is the hardest. In
fact, since there is some time overhead spent in absorbing and understanding the
game statement, we may welcome a (simple) seven question game, since we can knock
out more questions without having to stop and absorb a new game statement.
The questions posed after the game statement loosely follow a general pattern. The
first couple of questions usually do not introduce any modification to the rules. This
happens with the later questions. Often one of the initial questions is a so-called
“acceptability” question”, which is designed to fritter away your valuable minutes by
requiring you to do a process of elimination. We have a suggestion later on how to
handle these efficiently. The flip side of the time-waster is that it may reinforce the
game statement in your mind so that you have a better perspective for the remaining
questions.
Questions posed after the first few often introduce special local or ad hoc
assumptions that change the rule environment. Generally you need to make secondary
deductions from the rules in order to answer these confidently. Certain types of
these secondary deductions are automatic, and we will discuss them later. Oddly or
not, very often the last question for a game has a “silver bullet” answer, by which we
mean it can be answered immediately without any lengthy process of elimination by
appealing to a single insight gotten from the rules and secondary deductions.
Provided that all the rules, modifications included, are identical for two questions, a
correct answer for one may be an immense help in answering the other, so don’t treat
the questions in isolation....there are linkages. And the linkages work both forwards
and backwards.
The LSAT authors adhere to some conventions in framing questions, so we will explore
them below. Also, we present a basic review of logic with particular emphasis on some
of the logical fallacies that test writers rely on for temptation.
QUESTION STYLES
Most logic game questions, adjusted for specifics, are of three types:
This is the most common type of question. To answer it you must be able to
make an argument that a particular configuration of variables is forced to
happen by the rules.
This is the flip side of type 1). We are now looking for what cannot possibly
happen. Don’t make the mistake of interpreting it as “could be false”...it is
definitely false. You could approach a question of this type by trying to
eliminate the four incorrect responses that “could be true”, but unless you have
some substantial information on this already, you are headed for wasting time
with a process of elimination. One nice thing about so-called counterexample
problems is that all it takes is for one thing to be out of place...one rule
violated...and you have your answer.
Question types 4) and 5) are clearly equivalent to 3). The test authors capitalize
words like “cannot” and “except” as a courtesy to the test taker, so there is no doubt
about the negation in the question.
7) Which of the following variables must come before a given variable (in a sequence)?
Question types 1) and 3) are generally easier to answer than type 2) because they
involve definite conditions. A solution template may rule out or rule in a particular
outcome, and that is enough to decide questions of those types. A question of type 2)
may require that we investigate each candidate statement and determine if it is not
necessarily false. Sometimes there may not be enough information contained in the
scenario and rules to reason your way to a definite answer for a type 2) question. In
that case, a trial solution diagram that can be shown not to violate any of the rules,
and therefore result in a “could be true” may be the only approach. This happens, so
don’t be discouraged by the apparent lack of definiteness.
For a type 1) or 3) question that does not yield easily to a direct interpretation, a
process of elimination, like that for a type 2) question, may have to be used, but try
to avoid it if at all possible. To rule out an answer to a “must be true” or “CANNOT
be true” question, it suffices to find a single respective counterexample. Don’t forget
that if you are reasonably sure that you have answered a type 1) or 3) question
correctly, the content of your answer is as good as another rule in guiding your
response to further questions based on the same rule set.
A type 6) question can be answered with the help of answers to type 1) questions. The
latter establish what is forced to be true, and that is the point of type 6) questions.
Type 7) and 8) questions also benefit from having as complete a picture as possible of
what is necessarily true. A type 9) question introduces a local rule...good just for that
question or maybe the one right after it, too. Local rules can turn “could be trues” into
“must be trues” by eliminating all but one rule-consistent variable configuration. So
having a confident answer to a “could be true” might lead to a quick answer for a type
9).
It is always a good idea to “pick the low hanging fruit”...by which we mean writing down
for easy reference the obvious implications of the scenario and rules. Let us call these
derived rules. As you parse the game statement, it may be immediately obvious that
a potentially useful derived rule can be inferred from the stated rules. Write this
down on speculation even before you approach the task of building a solution diagram.
You may need it...maybe not...but very often the low fruit leads to an easier time with
the questions. If nothing occurs to you, don’t try to force it...concentrate on the
questions.
We can characterize the two events in the following way. A is a sufficient condition
for B, since whenever we have A, we then have B. Not only do we just have B, we must
have B, and that means B is a necessary condition for A. You can’t have A without B.
Logicians call a conditional statement like this a material implication. It is a weaker
assertion than it may first appear to be. The only thing that is being ruled out is that
you can’t simultaneously have A and not B.
We have squeezed all the juice out of the conditional statement “If A, then B”. A is
sufficient for B and B is necessary for A. A common mistake is to think that if B has
happened, that must mean A has happened, or that B is sufficient for A. It often
seems like this is a superficially reasonable conclusion, especially if the most probable
way for B to happen is to be preceded by A, but it does not follow from the rules of
logic. “If you stay out too long in the sun, then you will have reddened skin.” Our
common experience suggests that reddened skin is often caused by too much sun, but
there could be other reasons...allergies, chemicals, or tanning beds, for example. “If
B, then A” does not follow from “If A, then B”.
Given the statement “If A, then B”, the converse statement is “If B, then A”, the
inverse statement is “ If A, then not B”, and the contrapositive statement is “If not
B, then not A”. We have just noted that the truth of the converse statement is
independent of the truth of the original conditional statement. The inverse statement
is the denial of the original, so when the original is true, the inverse is false, and vice
versa. The contrapositive statement is true or false precisely whenever the original
conditional statement is true or false. We say that they are logically equivalent. Note
that in the original A is sufficient for B and B is necessary for A. So if we deny B,
that is sufficient to conclude that we cannot have A...but this is the contrapositive.
In the context of logic games, we are often given a rule such as “If A is in position 2,
then B is in position 5". We may not conclude that if B is in position 5, then A must be
in position 2. This is the converse. But we may conclude that if B is not in position 5,
then A is not in position 2. Again, this is the contrapositive. Get into the habit of
writing down the contrapositive for every “if-then” statement you see in a logic game.
The English language is very expressive, and there are many ways to make a
conditional statement. Some samples are presented below. We recommend converting
all conditionals to the model “if...then” form, which will be the most familiar to us.
If the furnace isn’t working, then the guest bedroom will be cold.
Unless the furnace works, the guest bedroom will be cold.
If the guest bedroom is not cold, then the furnace must be working.
Unless the guest bedroom is not cold, the furnace must not be working.
Should the furnace not be working, the guest bedroom will be cold.
In the event that the guest bedroom is not cold, then the furnace must be working.
Either the furnace is working or the guest room is cold.
Except when the furnace is working, the guest room is cold.
Whenever the guest room is not cold, the furnace must be working.
Every one of these statements is logically equivalent to the first, and hence to each
other. We will want to practice parsing conditional statements and reducing them to
the more familiar “if-then” form.
If A then B, and subsequently if also not A, then not B. This is called the fallacy of
denying the consequent. A happening definitely causes B to happen, but just because
A does not happen, B may still happen for some other reason.
If Johnson pitches, then we will win. Johnson didn’t pitch. We won anyhow.
If A then B, and subsequently if also B, then A. This is called the fallacy of affirming
the antecedent. Again, A happening forces B to happen, but if B happens
spontaneously, that doesn’t mean A had to happen.
If it rains, the street will be wet. The street is wet, therefore it must be raining.
Could have been a fire hose!
MODELING LOGIC GAMES
NOTATION
A good diagram or template that faithfully models the scenario and rules of a game
statement is priceless. That is because it summarizes visually all of the quirks of a
game and allows you to see subtleties and relationships that might not be apparent
from simply reading the words of a game statement. A good diagram may answer some
questions all by itself. Several observations can be made about “optimal notation”:
1) You have to be comfortable using it...it has to be natural for you and effortlessly
recalled. Some of the national prep courses have complicated systems where learning
the notational system is a project in itself. You will be under constant time pressure
taking the exam, and mentally fumbling for the niceties of some notational system is
not what you want to be doing in the middle of the exam. Recall the story of the
medical student who had studied a mnemonic phrase (little ditty that spurs the
memory) to remember the Latin names for the craniofacial nerves. He took his exam
and got so confused with the ditty that he completely botched the list of nerves.
2) It has to be sketched quickly. Given enough time, we could probably create a game
diagram that flawlessly and completely summarized anything we might ever want to
know about the game. But we don’t have unlimited time, and the important thing is to
get something written quickly and accurately that will give us the insights we need.
3) It has to fit legibly in the space we have to write it. Learn to write small. Scratch
work has to be done on the examination pages and there is not a lot of room.
Questions that introduce new or modify old rules may require a re-do of the template.
Or you may get off on the wrong foot and have to start over on a diagram. Space is
at a premium as well as time. We recommend committing to a horizontal format for
whatever diagram you develop. The text on the pages is horizontal, and you will be
able to scale and position your scratchwork better.
4) It has to be expressive enough to model the features of every game you might
encounter.
The following chart presents a notational system that we have found satisfies the
above desirable properties. This is admittedly subjective, so we present our system
as a starting point for you.
ST S immediately precedes T
...S
_... S must be at position n
n
..._
S/... S could be at position n
n
nS S occurs n times
STRATEGY
GENERAL OBSERVATION
Linear games have historically, along with grouping games, been the bread-and- butter
puzzles for the LSAT. Rarely during the last twenty years has the LSAT not
contained at least one straight linear game per test date. Linear games are easy to
understand, if perhaps difficult at times to solve, so they lend themselves to the kind
of instructional approach we are trying to provide here. Certain principles of game
construction are used over and over again, so if we study these diligently, we will have
both a sense of familiarity with the type of game and enough confidence in our ability
to solve it that we may actually do so with a reasonable proportion of success.
RECOGNITION KEYS
The first task with linear games, as with any type of game, is to recognize them
correctly. The game principle is, of course, putting things in order or sequencing
them, so we should be on the lookout for clues that suggest this purpose in a game
statement. Words that indicate some variables are before or after others in time,
importance, or with respect to some other measure that reflects an intrinsic order
are the key.
The test makers may try to de-emphasize the nature of a linear game, but it is always
visible to the trained eye. The basic components are variables...objects that will be
placed in positions...and the positions available. A linear game statement will introduce
some kind of sequence of positions...days of the week, hours in a day, seats in a row,
rows in a stadium...that sort of thing. It may be far from time-related, and, in fact,
the sequence of positions could be based on something abstract, like consumer
preferences, or some other non-physical scale. But the defining feature is that
somehow the available positions will be ranked from first to last, lowest to highest,
worst to best, or by a similar type of order.
Sometimes the line between variables and positions is a little murky, and under the
battle conditions of an exam, there may be an opportunity for momentary confusion.
One way to avoid this is to note that in the rules for a linear game, almost always the
variables appear as subjects in the rule statements, and the positions available are
described in the predicates. Occasionally the test authors are completely
transparent about the nature of a linear game, and almost draw the game diagram for
you.
Now that we are sure we have a straight linear game, there are some features that
appear frequently.
GAME FEATURES
It is always clear how many positions are available for placing variables in order.
Sometimes a rule explicitly tells you that a particular variable must go in a particular
position. This is valuable, as it takes that variable off the table, so to speak. Use that
variable as an anchor to roughly locate other variables. For example, if A is in position
three out of five available, and B is later than A, then B must be in position four or
five.
A common rule in linear games is that either one variable immediately precedes
another, or it just precedes it, but we don’t know by how much. This immediately
precludes certain configurations of these variables. The variable that is “earliest”
cannot be last, as there would be no place for the “later” variable. Likewise, the later
variable cannot be first, as there would be no place for the earlier. If we know how
many positions separate two variables with this type of rule in effect, it may even be
enough to completely specify their positions in the overall sequence. For example, if
A must be two positions before B (there is one skipped position between them) and
there are only three positions available in this game, it is unavoidable that A will come
first and B last. Even if there were five positions available in this game, we could still
conclude that A would be restricted to the first, second, or third position, and
likewise B would only have access to the third, fourth, or fifth position. So a
seemingly simple relative relationship between two variables may have substantial
implications regarding their absolute placement.
Another trick the game authors use is to give a rule which pastes several variables
together into a block. Then they create an obstacle in a sequence that forces the
block, or something else that has to fit beside the block, into a definite absolute
position. For example, say we are to sequence A thru E in five positions. If A
immediately precedes B and B immediately precedes C, then somewhere the block ABC
must appear. If we now ask, if E must precede C, but D must follow A, the only game
in town is to put the block square in the center of the available positions, E first, and
D last. We go from seemingly innocent relative position rules to a hard and fast unique
final position of all variables. This is a gimmick that crops up often.
The preceding block trick can be expanded on by creating blocks of different sizes.
Then depending on the placement of the obstacle...a variable that by assumption goes
in a particular place...it may very well be that the remaining multiple-variable blocks
only fit into the grand scheme one way. Imagine a block of three and a block of two
variables to be placed in a sequence with six available positions. If the obstacle is
placed in the first or last position, there are two ways each to position the blocks. But
if the obstacle sits on the fourth position, there is a unique way to assign the blocks
to the sequence. If the obstacle were in the fifth position, there would be no
legitimate placement for the blocks available!
These are the main principles the game authors use to limit the options available in a
linear game without appearing to do so in the statement of the rules.
Linear Game 1
Scenario
A professor will listen to exactly one speech from each of six students – H, J, K, R, S
and T. The six speeches will be delivered one at a time, consecutively, according to the
following conditions:
Rules
1) The speeches delivered by H, J, and K, no matter what their order relative to each
other, cannot form a sequence of three consecutive speeches.
2) The speeches delivered by R, S, and T, no matter what their order relative to each
other, cannot form a sequence of three consecutive speeches.
3) H’s speech must be earlier than S’s speech.
4) J’s speech can be neither first nor sixth.
5) T’s speech can be neither immediately before nor immediately after J’s speech.
Questions
1) Which of the following could be the order, from first to last, in which the students
deliver their speeches?
(A) H, J, R, S, T, K
(B) H, R, T, K, S, J
(C) K, J, T, H, S, R
(D) R, J, K, T, H, S
(E) T, R, J, S, K, H
2) If T delivers the third speech, which one of the following must be true?
3) If S delivers the third speech and T delivers the fourth speech, then which one of
following must be true?
4) If K delivers the first speech and H delivers the fifth speech, which one of the following
must be true?
5) If R’s speech is immediately after S’s speech and immediately before K’s speech,
then which one of the following could be true?
6) If K delivers the third speech, any of the following could be the student who makes the
fourth speech EXCEPT
(A) H
(B) J
(C) R
(D) S
(E) T
Linear Game 1 - Analysis
1) Answer is (D)
By the process of elimination, we can rule out (A) as R, S, T cannot form three
consecutive speeches. Then we eliminate (B) as J cannot be sixth. Then we
eliminate (C) as JT cannot be consecutive. Finally, we can rule out option (E) as
H must be before S.
2) Answer is (B)
We can obtain this answer directly since if T is placed third, J cannot be second
or fourth. Taking this fact along with the original rules that J cannot be first or
sixth, we see that J must be fifth.
3) Answer is (C)
4) Answer is (A)
Placing K first and H fifth forces S into sixth place since H must be before S. The
rule that states T and J cannot be consecutive dictates that T cannot be third (as
that would leave only slots 2 and 3 would be available – neither of which J could
fill). Similarly J cannot be third. This leaves R to go in third place.
6) Answer is (A)
We can see that if we put H in position 4 that leaves no position available for J. And J
cannot be in position 1 or 6 anyway, now with K in position 3 and H in position 4, J
cannot occupy position 2 or 5 as that would create J, K, and H in a consecutive triple.
Linear Game 2
Scenario
Rules
1) J is not advertised during a given week unless H is advertised during the immediately
preceding week.
2) The product that is advertised twice is advertised during week 4, but is not advertised
during week 3.
3) G is not advertised during a given week unless either J or else O is advertised that week.
4) K is advertised during one of the first two weeks.
5) O is one of the products advertised during week 3.
Questions
(A) H and K
(B) H and M
(C) J and O
(D) K and L
(E) L and M
(A) G
(B) J
(C) K
(D) L
(E) M
4) Which one of the following CANNOT be the product that is advertised
during two of the weeks?
(A) G
(B) H
(C) K
(D) L
(E) M
(A) G
(B) H
(C) J
(D) K
(E) M
(A) H
(B) J
(C) K
(D) L
(E) O
(A) G and H
(B) H and J
(C) H and O
(D) K and O
(E) M and O
Linear Game 2 - Analysis
Immediate observation:
Note that the product advertised twice cannot be O, (since O is advertised in week 3 and
the repeated product is advertised in week 4 but not advertised during week 3), it also
cannot be J (since if J were advertised twice, then H would be advertised twice). Also, J
cannot be advertised during week 1 since J is only advertised when H is advertised the
previous week.
1) Answer is (B)
We can eliminate (D) since K must be advertised during week 1 or 2. We can also
eliminate (A) as J cannot be advertised during week 1. We can also rule out (C) as
G must appear with O or J. Finally we can eliminate (E) as J cannot be advertised
during week 2 unless H is advertised during week 1.
2) Answer is (C)
3) Answer is (B)
Note – this adds a new global rule: “J must be advertised during week 2” and hence
H must be advertised during week 1.
4) Answer is (A)
5) Answer is (E)
6) Answer is (D)
We know that J can only be advertised during week 2 so we can rule out (B).
Similarly, O can only be advertised during week 3 so we can rule out (E). The
product that is advertised twice cannot be advertised on week 3, so K and H cannot
be advertised on week 3. (H is advertised on week 1 and if it is advertised twice, it
will run on week 4 the second time around. K is advertised on either week 1 or
week 2, so if it is advertised twice it will run on week 4 the second time around).
This leaves L that can be advertised during any of the four weeks.
7) Answer Is (E)
We can rule out (A) because G must be advertised during the same week as either J
or O. We can rule out (B) since if H and J are advertised together it would have to be
during week 2 (this is the only week that J can be advertised), but if J is advertised
during week 2, then H must also be advertised during week 1. Thus making H the
product advertised twice, but this violates the rule that states that the duplicated
product must be advertised during week 4. We can rule out (C) for similar reasons. H
would have to be the duplicated product, yet isn’t being advertised during week 4.
Lastly, we can rule out (D) for the same reason. If K is the duplicated product, K must
be advertised on week 4, yet if K appears with O it would have to be during week 3,
and we already know that K is advertised during either week 1 or 2. The answer is
(E).
Linear Game 3
Scenario
A college offers one course in each of three subjects – mathematics, nutrition, and
oceanography – in the fall and again in the spring. Students’ book orders for these
course offerings are kept in six folders, numbered 1 through 6, from which labels
identifying the folders’ contents are missing. The following is known:
Rules
1) Each folder contains only the orders for one of the six course offerings.
2) Folder 1 contains orders for the same subject as folder 2 does.
3) The orders in folder 3 are for a different subject than are the orders in folder 4.
4) The fall mathematics orders are in folder 1 or else folder 4.
5) The spring oceanography orders are in folder 1 or else folder 4.
6) The spring nutrition orders are not in folder 5.
Questions
1) Which of the following could be the list of the contents of the folders, in order from
folder 1 to folder 6?
(A) fall mathematics, spring mathematics, fall oceanography, fall nutrition, spring
nutrition, spring oceanography.
(B) fall oceanography, spring nutrition, fall nutrition, fall mathematics, spring
mathematics, spring oceanography.
(C) spring mathematics, fall mathematics, spring nutrition, fall oceanography, fall
nutrition, spring oceanography.
(D) spring oceanography, fall oceanography, fall nutrition, fall mathematics, spring
mathematics, spring nutrition.
(E) Spring oceanography, fall oceanography, spring mathematics, fall mathematics,
fall nutrition, spring nutrition.
5) If the fall oceanography orders are in folder 2, then for exactly how many of the
remaining five folders can it be deduced which course offering’s orders are in that
folder?
(A) One
(B) Two
(C) Three
(D) Four
(E) Five
6) Which one of the following lists a pair of folders that must together contain orders for
two different subjects?
(A) 3 and 5
(B) 4 and 5
(C) 3 and 6
(D) 4 and 6
(E) 5 and 6
7) Which one of the following could be true?
(A) The fall mathematics and spring oceanography orders are in folders with
consecutive numbers.
(B) Folder 5 contains the orders for a spring course.
(C) Folder 6 contains the orders for a subject other than nutrition.
(D) The mathematics orders are in folders 1 and 4.
(E) The orders for the fall courses are in folders 1, 3, and 6.
Linear Game 3 - Analysis
1) Answer is (D)
By process of elimination it cannot be (A) since the fall nutrition orders cannot
be in folder 4. It cannot be (B) or (C) as either the fall mathematics orders or the
spring oceanography orders must be in folder 1. It cannot be (E) as folders 3
and 4 cannot both contain the mathematics orders.
2) Answer is (A)
We can reach this solution directly, since if the fall mathematics orders are in
folder 1, the spring mathematics orders are in folder 2. Alternatively, if the fall
mathematics orders are in folder 4, the spring mathematics orders cannot be in
folder 3. Note – this is a new global rule: Spring mathematics orders are not in
folder 3.
3) Answer is (B)
We can automatically see that (A), (C), (D), and (E) are false (cannot be true
since if the fall oceanography orders are in folder 2, then the spring
oceanography orders are in folder 1, and so the fall mathematics orders must
be in folder 4. It follows that folder 3 contains a nutrition order.
4) Answer is (D)
Process of elimination: (A) and (B) are false as folder 1 contains either the
mathematics orders or the spring oceanography orders. (C) is false as folder 4
contains either the fall mathematics orders or the spring oceanography orders.
is false since if the spring oceanography orders are in folder 1, then the fall
oceanography orders are in folder 2 and the fall mathematics orders are in
folder. But folder 3 contains orders for a different subject than folder 4, which
means it must contain either the fall or the spring nutrition orders.
5) Answer is (B)
Since folders 1 and 2 have orders for the same subject, we have only the
orders for two subjects left that could be in folders 3 – 6. Folders 3 and 4
contain different subjects (one of each) which means folders 5 and 6 must also
contain one of each.
7) Answer is (C)
By process of elimination we see that (A) is false since the fall mathematics
orders and the spring oceanography must be in folders 1 and 4. (B) is also false
as the spring nutrition orders cannot be in folder 5 and neither can the
oceanography orders. (D) is false as the spring oceanography orders must be
in either folder 1 or folder 4. (E) also has to be false, since if the fall
mathematics orders are in folder 1, the fall nutrition orders are in folder 3, and
the fall oceanography orders are in folder 6, then the spring nutrition orders
would have to be in folder 5 and this cannot be.
Linear Game 4
Scenario
On a particular Saturday, a student will perform six activities- grocery shopping, hedge
trimming, jogging, kitchen cleaning, laundry, and motorbike servicing. Each activity will
be performed once, one at a time. The order in which the activities are performed is
subject to the following conditions:
Rules
Questions
1) Which of the following could be the order, from first to last, of the student’s activities?
5) Which one of the following, if substituted for the condition that motorbike servicing
has to be earlier than laundry, would have the same effect in determining the student’s activities?
1) Answer is (D)
2) Answer is (B)
Only one of the five CANNOT be third. That means the others could possibly be
third. The rule that states that K < HG is the tightest rule so it is a good idea to
start with that. If hedge trimming is third, then it follows that grocery shopping is
fourth and so motorbike servicing and jogging must either be in positions 5 and
6, or positions 1 and 2 (since they must be done consecutively). However neither
of these will work because of the fact that laundry must be done after motorbike
servicing (which means that motorbike servicing and jogging cannot be done fifth
and sixth), and also kitchen cleaning must be done before grocery shopping
which would mean that kitchen cleaning must be done first or second, yet
positions 1 and 2 are taken up by motorbike servicing and jogging. Note – this
provides a new GLOBAL RULE: Hedge trimming cannot be done third. It also
follows that grocery shopping cannot be done fourth.
3) Answer is (C)
If kitchen cleaning is done second, then position 1 is unusable as the only other
‘single’ task (ie- one that doesn’t have to be done consecutively with another
task) is laundry, yet laundry must be done after motorbike servicing, so laundry
cannot be done first. This is a new GLOBAL RULE: Kitchen cleaning cannot be
done second.
4) Answer is (D)
If laundry is done fifth, then position 6 is unusable as the only other ‘single’ task
(ie- one that doesn’t have to be done consecutively with another task) is kitchen
cleaning, but that task must be done prior to grocery shopping. This is a new
GLOBAL RULE: Laundry cannot be done fifth.
5) Answer is (C)
This is the only option that doesn’t completely change the rule: M,J < L (Or J, M
< L).
Linear Game 5
Scenario
In the course of one month Garibaldi has exactly seven different meetings. Each of her
meetings is with exactly one of five foreign dignitaries: Fuentes, Matsuba, Rhee,
Soleimani, or Tbahi. The following constraints govern Garibaldi’s meetings:
Rules
1) She has exactly three meetings with Fuentes, and exactly one with each of the other
dignitaries.
2) She does not have any meetings in a row with Fuentes.
3) Her meeting with Soleimani is the very next one after her meeting with Tbahi.
4) Neither the first or last of her meetings is with Matsuba.
Questions
1) Which of the following could be the sequence of the meetings Garibaldi has with the
dignitaries?
2) If Garibaldi’s last meeting is with Rhee, then which one of the following could be
true?
3) If Garibaldi’s second meeting is with Fuentes, then which one of the following is a
complete and accurate list of the dignitaries with any one of whom Garibaldi’s fourth
meeting could be?
4) If Garibaldi’s meeting with Rhee is the very next one after Garibaldi’s meeting with
Soleimani, then which of the following must be true?
5) If Garibaldi’s first meeting is with Tbahi, then Garibaldi’s meeting with Rhee could be
the
6) If Garibaldi’s meeting with Matsuba is the very next meeting after Garibaldi’s meeting
with Rhee, then with which one of the following dignitaries must Garibaldi’s fourth
meeting be?
(A) Fuentes
(B) Matsuba
(C) Rhee
(D) Soleimani
(E) Tbahi
Linear Game 5 - Analysis
1) Answer is (C)
This is a typical first question, asking for a sequence that violates no rules.
Use process of elimination by appealing to the basic rules. First check for three
meetings with Fuentes. (A) is eliminated but no others. Check for consecutive
meetings with Fuentes. (B) is eliminated but no others. Check for TS
(Tbahi/Soleimani). (D) fails, but no others. It is now between (E) and (C). Check
the Matsuba rule - can’t be first or last - that eliminates (E).
2) Answer is (D)
This question introduces a local rule - Rhee is last - so we can make a copy of
our basic diagram and put R in last. We scan our rules to see if this forces any
other positions, but it doesn’t do so obviously. There is too much apparent
freedom. We suspect at this point that there are several legitimate solutions with
Rhee last, but what will happen is that only one answer will correspond to a
legitimate solution. Looks like this question may be a time eater. If we are
confident, we can just start plugging in letters, starting with the one given in each
answer, and see what fails. If we are lucky and one works, we are done.
If you invested three minutes in this question with no progress, drop it and move
on. Remember...even eliminating one answer improves your odds for a correct
guess.
3) Answer is (E)
Draw a secondary sketch with F in the second position. Ask who can go first.
Certainly not another F, since FF is out. M is already excluded. The TS condition
rules out either T or S. So evidently it is R that must go first. Fill that in. Which
answers are ruled out by that..(A), (B), and (D). The remaining choices both have
S as a possibility, so that must be true. So it comes down to either M or F being
the other possibility for fourth position. Recall that we have to place two more F’s
and they cannot be adjacent. If M were fourth, the TS pair would have to be after
it, then one of the remaining two F’s would have to be in third position. This puts
two F’s together, which can’t be. Apparently F must go in position 4.
4) Answer is (E)
Draw another secondary sketch and add the local rule SR. Since we already
have TS, make it TSR. This has to appear somewhere as a block. Test it in
various positions and watch where the F’s could go...recall they must be
separated. The only things available for this purpose are the block TSR and the
single letter M. The only way they can be used as separators is if there is an F in
positions 1 and 7. The only sure bet among the answers is then (E).
5) Answer is (D)
Draw yet another secondary sketch and put T first. The TS condition (this is
global...applies to entire problem...unlike the TSR condition which was just for the
preceding question) requires S to be second. The only feasible placement of F’s
is in slots 3,5, and 7. This leaves R and M for slots 4 and 6. Only R being sixth is
a choice.
6) Answer is (A)
Draw another secondary sketch and add RM as a local rule. We already have
TS, and the three F’s must be separated by these two double letters. This forces
the F’s to be in slots 1,4, and 7, so (A) is the only one that works.
Linear Game 6
Scenario
Three short seminars - Goals, Objections, and Persuasion - and three long seminars -
Humor, Negotiating, and Telemarketing - will be scheduled for a three day sales training
conference. On each day, two of the seminars will be given consecutively. Each seminar
will be given exactly once. The schedule must conform to the following conditions:
Rules
1) Exactly one short seminar and exactly one long seminar will be given each day.
2) Telemarketing will not be given until both Goals and Objections have been given.
3) Negotiating will not be given until Persuasion has been given.
Questions
1) Which one of the following could be an accurate schedule for the sales training
conference?
2) If Goals is given on the first day of the sales training conference, then which one of
the following could be true?
4) Which of the following CANNOT be the second seminar given on the second day of
the sales training conference?
(A) Humor
(B) Persuasion
(C) Objections
(D) Negotiating
(E) Goals
5) If Humor is given on the second day of the sales training conference, then which one
of the following could be true?
1) Answer is (B)
This is a typical first question, asking for a sequence that violates no rules.
Use process of elimination by appealing to the basic rules. (A) puts g after T
contrary to the second rule, (B) doesn’t seem to violate any rules, (C) puts p
after N, contrary to the third rule, (D) puts g and o on the first day, violating the
first rule, and (E) places o after T, again contrary to the second rule.
2) Answer is (E)
Draw a secondary sketch with g in the first position. The choices for second
position then are H, N, and T. It cannot be N, which has to come after p. It
cannot be T, since that has to come after o. It must therefore be H. Then the
sequence starts with g and H, and every answer except E conflicts with this.
3) Answer is (C)
Add the local rule N < o. Combining this with our other before/after rules, we get
the following derived rule: p < N < o < T. (A), (B), (D), and (E) are all out since
g,H,p,N,o,T satisfies the rules. This leaves (C).
4) Answer is (B)
If we take our own advice to look over and attempt the “could be true” questions
first, we would have temporarily skipped over this one and done the next
problem. The statement of problem 5) tells us immediately that answer (A) for
problem 4) is wrong. The sketch for problem 5) reveals that g or o could be on
the second day, so answers (C) and (E) are out as well. Now we are down to (B)
or (D). If p is in fourth position (second seminar of second day), then the p < N
rule puts N on day 3. This means T is the first seminar on day 2, since it can’t be
on day 1. But both g and o must precede T, and there isn’t room for two short
seminars before T. So this configuration cannot work and (B) is the correct
response.
5) Answer is (D)
Draw another sketch with H on the second day. Consider where N and T can
appear. T is certainly not on day 1, so the only possibility is that appears on day
3. That leaves N on day 1. Now place g, o, and p. We know p is before N, so p
must go in the very first position with N second. The g,o < T rule allows g or o to
be on day 2 (third or fourth position) or before T on day 3 (fifth position). Looking
at the answer choices, (A) and (C) are out because T can’t be on day 1. (B) is out
because we needed N on the first day (in position 2). (E) is out because it has to
precede N. The only remaining choice is (D).
Linear Game 7
Scenario
Each of exactly three actors - Gombrich, Otto, and Raines - auditions for parts on
exactly two of the following days of a particular week: Wednesday, Thursday, Friday,
and Saturday. On each of these days at least one of the actors auditions for parts. The
order of that week’s auditions must meet the following conditions:
Rules
1) The first day on which Otto auditions is some day before the first day on which Raines
auditions.
2) There is at least one day on which both Gombrich and Raines audition.
3) At least one of the actors auditions on both Thursday and Saturday.
Questions
1) Which of the following could be an accurate matching of the actors to the days on
which they audition?
2) If Otto auditions on both Thursday and Saturday, then Gombrich could audition on
both
4) Which one of the following pairs of days CANNOT be the two days on which Otto
auditions?
6) If Gombrich auditions on both Wednesday and Saturday, then which one of the
following could be true?
1) Answer is (B)
The rule O1 < R1 eliminates answer choices (C), and the rule that we must have
G and R together on one of the days eliminates (E). We can eliminate (A) as
none of the actors audition on both Thursday and Saturday. Finally we can
eliminate (D) as no one auditions on Friday, and at least one of the actors must
audition on each day.
2) Answer is (B)
Since O auditions on Thursday and Saturday, and R must audition at least one
day after O first auditions, we can conclude that R auditions on Friday and
Saturday. Now since G must audition with R on at least one of the days, and
someone needs to audition on each day, G must audition on Wednesday and
Friday.
3) Answer is (E)
E must be false since if R’s last audition is on Thursday, then R must audition on
Wednesday and Thursday, which means that R cannot audition earlier than R as
required. This is an example of how reading all five answer choices first can save
you a lot of time!
4) Answer is (D)
(D) and (E) are the most restrictive of the five choices, since once O1 is
positioned for Thursday, R must audition on Friday and Saturday. In fact, if O
auditions on Thursday and Friday, R must audition on Friday and Saturday, but
then G must also audition on Saturday since we have to have at least one day
where R and G audition together. But this means that G must audition on
Thursday also (with O) since one of the actors must audition on Thursday and
Saturday, however this means that no one is auditioning on Wednesday which
violates the rules.
5) Answer is (C)
Scenario
Rules
Questions
1) Which of the following could be the order of the programs, from most popular to least
popular?
(A) J, H, L, Q, V, S, P
(B) H, L, Q, J, S, P, V
(C) H, J, Q, L, S, V, P
(D) H, J, V, L, Q, S, P
(E) H, L, V, J, Q, P, S
2) If J is more popular than L, and S is more popular than P, then which one of the
following must be true of the ranking?
(A) J is second
(B) J is third
(C) L is third
(D) Q is third
(E) P is seventh
(A) L
(B) J
(C) Q
(D) V
(E) P
4) If V is more popular than Q and J is less popular than L, then which one of the
following could be true of the ranking?
5) If Q is more popular than L, then each of the following must be true of the ranking
EXCEPT:
(A) H is first
(B) L is fourth
(C) V is not fourth
(D) J is not third
(E) Q is third
Linear Game 8 - Analysis
1) Answer is (C)
From the tree diagram we see that H is the most popular so we can eliminate
choice (A). Choice (B) is incorrect since J is more popular than Q. (D) is wrong
as L is more popular than V, and finally (E) is false since we know that S is not
ranked seventh.
2) Answer is (A)
3) Answer is (E)
From the original tree diagram we see that P is less popular than H, J, and Q
and so cannot be ranked third.
4) Answer is (D)
We can eliminate (B), (C), and (E) using the diagram, so initially it looks like both
(A) and (D) could be possible – however we know that S is not seventh, so S
must be more popular than P
5) Answer is (B)
From the tree diagram we can see that (A), (C), (D), and (E) must be true, which
leaves (B) that isn’t necessarily true.
GROUPING GAMES
STRATEGY
GENERAL OBSERVATION
Grouping games are the other mainstay of the LSAT besides linear games. Where
linear games emphasize order and require that variables be placed in a sequence
conforming to the rules, grouping games are based on placing variables into categories
that likewise obey some conditions. If you are familiar with the ideas of sets and
subsets, you can think of the defining task of a grouping game as the creation of
subsets of variables under certain restrictions. We don’t have the before/after
concept in grouping games, so we have to give up some of the automatic secondary
deductions we used for linear games. But the good news is that there are replacement
deductions of this sort available for grouping games built around the ideas of “must
be with” and “must not be with”.
RECOGNITION KEYS
As we have mentioned, the very first task is to properly recognize the type of game
you are dealing with. You have studied the major game types and have a battle plan for
each, so it is important to avoid getting off on the wrong foot. With a pure grouping
game, you should not be seeing words in the scenario or rules that have anything to
do with sequencing. Instead there will words and phrases that indicate objects must
be categorized, or put into classes, or aggregated some way...or other phrasing
conveying the same effect. When you see rules that specify some variables must
appear, can appear, or cannot appear with others, it is a safe bet that you have a
grouping principle at the root of the game. Other constraints might be phrased
conditionally, for example, “If A is in the first group, then B must be in the third
group”.
Grouping games are more general than linear games in the following sense. The
ordering property in linear games has only so many ways of being expressed
realistically. After time, spatial position, and various artificial rankings (gold, silver,
bronze, etc.), there aren’t too many sequencing principles that could be the basis for
a realistic game. In contrast, grouping games can be framed in terms of just about
anything that can be sorted.
Our advice for detecting complex games holds here as well. If you see the grouping
game language above in the scenario and rules...but you also see references to some
sequencing activity...you have a hybrid or combination game. A pure grouping game is
usually simpler than a combo game, so you may want to attack the pure linear and pure
grouping games first. The test makers typically give you at least one of each.
Once you know you have hooked a grouping game, there are several standard moves
to make in setting up your analytical diagram.
GAME FEATURES
Continuing with the preceding example, we can illustrate another principle that is
frequently encountered in grouping games. Some forms of ambiguity never have to be
totally sorted out to get a correct answer. If we imposed only the rule that no
category can be empty, a good test question would be “How many distinct
configurations are possible with exactly two variables in either category?” There are
six. It isn’t necessary to write them all out (the example is so small that this is not
impractical). If the distribution is always two in one and one in the other, then either
A, B, or C can be alone in either category 1 or 2. These would all be different
configurations, and so there is your six.
Occasionally it is possible, and always desirable, to link together rules that restrict
the same variable. Perhaps you have noticed that in game statements some variables
seem to pop up in the rules more than others. In our example again, if A must be in
category 1 and B cannot appear together with A, then we immediately splice those
rules together in a secondary deduction: B must be in category 2. Of course this is
simplified just for the purposes of illustration, but quite a lot of valuable deductions
can be teased out of combinations of grouping rules. Focus on the highly restricted
variables. And if you see a conditional “if-then’, be sure to write down the
contrapositive so it is on the radar.
Grouping Game 1
Scenario
Each of five students – Hubert, Lori, Paul, Regina, and Sharon – will visit exactly one of
three cities – Montreal, Toronto, or Vancouver – for the month of March, according to the
following conditions:
Rules
Questions
(A) Hubert, Lori, and Paul visit Toronto, and Regina and Sharon visit Vancouver.
(B) Hubert, Lori, Paul and Regina visit Montreal, and Sharon visits Vancouver.
(C) Hubert, Paul, and Regina visit Toronto, and Lori and Sharon visit Montreal.
(D) Hubert, Regina, and Sharon visit Montreal, and Lori and Paul visit Vancouver.
(E) Lori, Paul, and Sharon visit Montreal, and Hubert and Regina visit Toronto.
2) If Hubert and Sharon visit a city together, which one of the following could be true in
March?
3) If Sharon visits Vancouver, which of the following must be true for March?
5) If Regina visits Toronto, which one of the following could be true in March?
1) Answer is (C)
We can rule out (A) since H and R must visit the same city. We can also rule out
(B) as the grouping must be 3/2 and this is 4/1. We eliminate (D) since L cannot
visit Vancouver, and finally it cannot be (E) because S and P cannot visit the
same city.
2) Answer is (D)
Since this is a “could be true” type of question, we can start by eliminating the
answer choices that “CANNOT be true”. Note that the two groups must be:
H/S/R (3) and L/P (2) since H and R must visit the same city. Knowing this set up
eliminates (A), (B), and (C) right away. We can also rule out (E), since L does not
visit Vancouver.
3) Answer is (D)
4) Answer is (A)
5) Answer is (C)
This is a “could be true” question, so we need to eliminate the four that are false.
(A) and (B) cannot be true since L must visit Montreal. (D) is false for two
reasons – If P visits Vancouver then so does H, but H is visiting Toronto. Also, L
cannot visit Vancouver.
6) Answer is (E)
Vancouver is a good starting point since we have two rules involving it. Since
Lori cannot visit Vancouver, neither does her partner (she is part of a double
set). This means that the triple must visit Vancouver.
Grouping Game 2
Scenario
Rules
Questions
(A) F, G, H, and J
(B) F, H, L, and M
(C) F, J, K and L
(D) G, H, L, and N
(E) G, K, M, and N
(A) G is in group 1.
(B) H is in group 2.
(C) J is in group 1.
(D) K is in group 2.
(E) M is in group 1.
(A) G is in group 2.
(B) J is in group 1.
(C) K is in group 1.
(D) L is in group 2.
(E) M is in group 2.
4) If L and M are in group 2, then a person who could be assigned to either group 1 or,
alternatively to group 2, is
(A) F
(B) G
(C) H
(D) J
(E) K
5) Each of the following is a pair of people who could be in group 1 together EXCEPT
(A) F and G
(B) F and H
(C) F and L
(D) H and G
(E) H and N
6) If L is in group 2, then each of the following is a pair of people who could be in group
1 together EXCEPT
(A) F and M
(B) G and N
(C) J and N
(D) K and M
(E) M and N
Grouping Game 2 - Analysis
1) Answer is (D)
F and J must be in the same group, so this excludes (B). G and M cannot
be together, this excludes (C) and (E). Finally, (A) is false since if H is in
group 1, then L must be in group 1.
2) Answer is (B)
3) Answer is (C)
Since we have F, J, and H together in one of the groups, the remaining
member must be G or M (as G and M cannot be in the same group).
Furthermore, this group must be group 2 since if H were in group 1, then L
would have to be in group 1 also. Thus it is clear that K, L, and N must be
in group 1 with either G or M. The only certainty of the five answer choices
is that K is in group 1.
4) Answer is (E)
L in group 2 implies that H is in group 2 (this is the contrapositive of rule
3). There is no room left in group 2 for F and J, so they must be in group
1, with G (to keep G and M separated). This set-up rules out (A) – (D) and
K is the only consumer who can be placed into either group 1 or group 2.
5) Answer is (B)
It is a good idea to start with answer choice (B) since we have a rule that
pertains to if H is in group 1. We also know that F and J must be together.
If F and H and together in group 1, J and L must be there also. However,
this means that G and M would be together in group 2 which cannot
happen.
6) Answer is (D)
If L is in group 2, then H is in group 2. Either G or M must be in group 2
(the other must be in group 1 with F and J). If K and M are together in
group 1, G must be in group 2, but G in group 2 means that N must be in
group 1 – but there is no more room as group 1 already contains F, J, K,
and M.
Grouping Game 3
Scenario
A reporter is trying to uncover the workings of a secret committee. The committee has
six members – French, Ghauri, Hsia, Irving, Magnus, and Pinsky – each of whom serves
on at least one subcommittee. There are three subcommittees, each having three
members, about which the following is known.
Rules
Questions
1) If French does not serve on any subcommittee with Magnus, which one of the
following must be true?
(A) Magnus serves on every subcommittee on which French serves and every
subcommittee on which Ghauri serves.
(B) Magnus serves on every subcommittee on which Hsia serves and every
subcommittee on which Irving serves.
(C) Hsia serves on every subcommittee on which French serves and every
subcommittee on which Ghauri serves.
(D) French serves on every subcommittee on which Pinsky serves.
(E) Hsia serves on every subcommittee on which Pinsky serves.
3) If Irving serves on every subcommittee on which Magnus serves, which one of the
following could be true?
1) Answer is (C)
Since F cannot serve with M, this means that M is not the triple, and so it must
be P. P being the triple means that P serves with everybody. In particular, P
serves with I. So answer choice (C) must be true.
2) Answer is (C)
3) Answer is (B)
If M always serves with I, then M cannot be the triple (as there is only 1), which
means that P is the triple. Similarly, M cannot be the double as if he were, then I
would also be a double (and there is only 1). This rules out (A) and (C). (D) and
(E) are false since M only serves once and that is with P and I. The correct
answer is (B) as I can serve twice. (M serves implies I also serves, but I can
serve without M).
4) Answer is (D)
We know that either M or P is the triple, so (A) and (B) are false. (C) is also false
since either P or M is the triple, it would mean that G is also a triple and there is
only one. (D) is the correct answer by process of elimination. (E) is also false as
M and P cannot both be triples which means that either M serves without P on at
least one subcommittee, or P serves without M on at least one subcommittee.
5) Answer is (E)
Since either M or P is the triple, they must serve together on at least one
subcommittee. We can use our previous work and diagrams to eliminate the
other answer choices. For example, we saw in question 19 that it is possible for I
to serve twice, thus (B) is false. The others can be eliminated in a similar way.
6) Answer is (D)
This is one of the original rules that we determined in the beginning. Look out for
simple questions right at the end of a more difficult logic game.
Grouping Game 4
Scenario
Each of seven travelers – Norris, Oribe, Paulsen, Rosen, Semonelli, Tan and
Underwood – will be assigned to exactly one of nine airplane seats. The seats are
numbered 1 through 9 and arranged in rows as follows:
Front row: 1, 2, 3
Middle row: 4, 5, 6
Last row: 7, 8, 9
Only seats in the same row as each other are immediately beside each other. Seat
assignments must meet the following conditions:
Rules
Questions
1) Which one of the following is a pair of travelers who could be assigned to seats 2
and 8 respectively
2) If Semonelli and Underwood are not assigned to seats in the same row as each
other, which of the following must be false?
(A) Oribe
(B) Rosen
(C) Semonelli
(D) Tan
(E) Underwood
4) If the seat to which Tan is assigned is immediately beside a seat assigned to another
traveler and also immediately beside one of the unassigned seats, which one of the
following must be true?
5) If Oribe is assigned to a seat immediately beside one of the unassigned seats, which
of the following must be true?
1) Answer is (C)
We can rule out (A) since H and R must visit the same city. We can also rule out
(B) as the grouping must be 3/2 and this is 4/1. We eliminate (D) since L cannot
visit Vancouver, and finally it cannot be (E) because S and P cannot visit the
same city.
2) Answer is (D)
Since this is a “Must be True” type of question, we can start by eliminating the
answer choices that “cannot be true” and “aren’t necessarily true”.
Note that the two groups must be: H/S/R (3) and L/P (2) since H and R must visit
the same city. Knowing this set up eliminates (A), (B), and (C) right away. We
can also rule out (E), since L does not visit Vancouver.
3) Answer is (D)
4) Answer is (A)
5) Answer is (C)
This is a “could be true” question, so we need to eliminate the four that are false.
(A) and (B) cannot be true since L must visit Montreal. (D) is false for two
reasons – If P visits Vancouver then so does H, but H is visiting Toronto. Also, L
cannot visit Vancouver.
6) Answer is (E)
Vancouver is a good starting point since we have two rules involving it. Since
Lori cannot visit Vancouver, neither does her partner (she is part of a double
set). This means that the triple must visit Vancouver.
Grouping Game 5
Scenario
In Crescentville there are exactly five record stores, whose names are abbreviated S, T,
V, X, and Z. Each of the five stores carries at least one of four distinct types of music:
folk, jazz, opera, and rock. None of the stores carries any other type of music. The
following conditions must hold:
Rules
Questions
(A) S carries folk and rock but neither jazz nor opera.
(B) T carries jazz but neither opera nor rock.
(C) V carries folk, rock, and opera, but not jazz
(D) X carries folk, rock, and jazz, but not opera.
(E) Z carries folk and opera but neither rock nor jazz.
3) If exactly one of the stores carries folk, then which one of the following could be true?
5) If V is one of exactly three stores that carry rock, then which one of the following
must be true?
6) If S and V both carry folk, then which one of the following could be true?
1) Answer is (E)
We can eliminate (A) since we know that S carries jazz. We can eliminate (B)
since we know T carries only opera and rock. (C) must be false since if V carries
folk, rock and opera, there are no types remaining for store Z. (Jazz is not an
option as only two of the stores carry jazz – and those stores are S and X). We
can eliminate (D) since X must carry all four types. (E) could be true.
2) Answer is (D)
We can eliminate (A) since V and Z cannot carry any of the same types. We can
eliminate (B) since only three of the stores carry jazz – S and X. We can
eliminate (C) since V does not carry jazz. Onl S and X do. Finally, we can
eliminate (E) since X must carry opera also.
3) Answer is (B)
If exactly one of the stores carries folk music, then we know that store must be
store X. This means that neither store V nor store Z carry jazz nor folk. Now
since that only leaves rock and opera and they cannot carry the same types of
music, V must carry rock or opera (not both) and Z carries the other (opera or
rock). In particular, stores V and Z only carry one type of music which eliminates
(A), (C), (D) and (E).
4) Answer is (C)
Since stores V and Z cannot carry the same types of music as each other, and
they cannot carry jazz, that leaves folk, opera and rock to stock between them
(but not all three need be stocked). This means store Z can carry at most two of
these, but at least one of these. This implies that store S (which carries three
types) must stock at least one more type than store Z.
5) Answer is (C)
If V carries rock, then Z cannot carry rock, and in fact Z must carry at least one of
folk and opera. Since T and X already carry rock, then none of the others (in
particular S), can carry rock. This means that S must carry folk, opera, and jazz.
Thus (C) must be true.
6) Answer is (B)
We know that S carries three types, two of which are folk and jazz. We also
know that T carries only rock and opera, and X carries all three. V carries folk
and possibly one other – either rock or opera (but not both since store Z must
carry at least one of rock/opera as well). This set-up means that (A), (C), (D) and
(E) are in fact false, but (B) is possible since Z could carry opera and S could
carry rock, jazz and folk.
Grouping Game 6
Scenario
There are exactly five pieces of mail in a mailbox: a flyer, a letter, a magazine, a
postcard, and a survey. Each piece of mail is addressed to exactly one of three
housemates: Georgette, Jana, or Rini. Each housemate has at least one of the pieces of
mail addressed to her. The following conditions must apply:
Rules
Questions
1) Which of the following could be a complete and accurate matching of the pieces of
mail to the housemates to whom they are addressed?
2) Which one of the following is a complete and accurate list of the pieces of mail, any
one of which could be the only piece of mail addressed to Jana?
4) Which one of the following CANNOT be a complete and accurate list of the pieces of
mail addressed to Rini?
5) If the magazine and the survey are both addressed to the same housemate, then
which of the following could be true?
1) Answer is (B)
Checking rule 1, (C) is out since Georgette can’t get the magazine. Rule 3 says
the flyer doesn’t go alone, so (E) is out. For rule 2 we look for Rini getting the
letter and Jana NOT getting the postcard. This eliminates (D). Notice that there
are only four pieces of mail delivered in (A), contrary to the scenario. (B) is left.
2) Answer is (B)
Georgette doesn’t get the letter, so either Rini or Jana gets it. If Rini gets it, then
rule 2 says Jana gets the postcard. Apparently Jana gets either the letter or the
postcard. The options besides (B) are all disqualified, since Jana cannot receive
only the survey or magazine (Rini would get the letter, forcing Jana to get at least
the postcard). Key concept here is “only” piece of mail.
3) Answer is (E)
(E) is immediate. If Jana doesn’t get the letter, then only Rini can, but then Jana
must get the postcard.
4) Answer is (B)
If Rini gets the letter, Jana must get the postcard. If Rini also gets the survey,
Jana, not Georgette, must get the magazine. Georgette must get something, and
it can only be the flyer. But the flyer cannot be the only piece of mail addressed
to Georgette. It follows that (B) is impossible.
5) Answer is (E)
It can’t be Georgette who gets the magazine and survey by rule 1. If the letter
goes to Rini, then Jana gets the postcard. Only the flyer is left, but it must be
accompanied by another item, so we can safely say that Jana gets the letter.
Now if Jana got the magazine and survey, in addition to the letter, the flyer goes
to someone else. But there aren’t enough items left to meet the rules (you would
need three). So it must be Rini who gets the magazine and survey, and Jana the
letter. Where can the flyer go? It could go to Jana, since she would have two
items. That would leave the postcard for Georgette. This configuration works,
and that is all we need.
Grouping Game 7
Scenario
A showroom contains exactly six new cars - T, V, W, X, Y, and Z - each equipped with at
least one of the following three options: power windows, leather interior, and sunroof. No
car has any other options. The following conditions must apply:
Rules
Questions
1) For exactly how many of the six cars is it possible to determine exactly which options
each one has?
(A) two
(B) three
(C) four
(D) five
(E) six
4) If Z has no options in common with T but has at least one option in common with
every other car, then which one of the following must be false?
5) Suppose that no two cars have exactly the same options as one another. In that
case, each of the following could be true EXCEPT:
6) If exactly four of the six cars have leather interiors, and exactly four of the six cars
have power windows, then each of the following must be true EXCEPT:
7) Suppose that the condition requiring that X has more options than W is replaced by a
new condition requiring that X and W have exactly two options in common. If all of the
other original conditions remain in effect, which of the following must be false?
1) Answer is (C)
From the diagram we see that the options for V,W, X, and Y have been
determined completely by the rules. We don’t know yet which one option T has,
nor which options Z has.
2) Answer is (A)
At most 4 of the cars can have leather interiors since we know that V and Y do
not have leather interiors.
3) Answer is (E)
Since Z only has two options, one of which is leather interiors, so the other must
be power windows and cannot be a sunroof.
4) Answer is (D)
If Z has at least one option in common with every other car, then Z must have a
sunroof – since Y has only a sunroof. It follows that T has power windows (since
T must have no options in common with Z). With this set-up, it is impossible for
exactly four of the cars to have leather interiors.
5) Answer is (C)
In this case, Z cannot have power windows, as it would have the same options
as W. Z must have a leather interior and a sunroof. This proves that at least four
of the cars (V, X, Y, and Z have sunroofs).
6) Answer is (D)
If exactly four of the six cars have leather interiors, then T must have leather
interiors. And if exactly four of the six cars have power windows, then Z must
have power windows. Now with this set up we see that W and Z have the same
two options.
7) Answer is (D)
The change in this rule only changes one thing in the original diagram – that X
may or may not have a sunroof. The only answer choice that must be false is
that X and Z have no options in common, since at the very least they both have
leather interiors. The other four choices all could be true.
Grouping Game 8
Scenario
In a repair facility there are exactly six technicians: Stacy, Urma, Wim, Xena, Yolanda,
and Zane. Each technician repairs machines of at least one of the following three types -
radios, televisions, and VCRs - and no other types. The following conditions apply:
Rules
Questions
1) For exactly how many of the six technicians is it possible to determine exactly which
of the three types of machines each repairs?
(A) one
(B) two
(C) three
(D) four
(E) five
(A) Of the types of machines reparied by Stacy there is exactly one type that
Urma also repairs.
(B) Of the types of machines repaired by Yolanda there is exactly one type that
Xena also repairs.
(C) Of the types of machines repaired by Wim there is exactly one type that
Xena also repairs.
(D) There is more than one type of machine that both Wim and Yolanda repair.
(E) There is more than one type of machine that both Urma and Wim repair.
3) Which one of the following must be false?
(A) Exactly one of the six technicians repairs exactly one type of machine.
(B) Exactly two of the six technicians repairs exactly one type of machine each.
(C) Exactly three of the six technicians repairs exactly one type of machine each.
(D) Exactly one of the six technicians repairs exactly two types of machines.
(E) Exactly three of the six technicians repairs exactly two types of machines
each.
4) Which one of the following pairs of technicians could repair all and only the same
type of machines as each other?
(A) There is exactly one type of machine that both Urma and Wim repair.
(B) There is exactly one type of machine that both Urma and Xena repair.
(C) There is exactly one type of machine that both Urma and Yolanda repair.
(D) There is exactly one type of machine that both Wim and Yolanda repair.
(E) There is exactly one type of machine that both Xena and Yolanda repair.
Grouping Game 8 - Analysis
1) Answer is (C)
From our drawing, the only technicians where the entire list of what they can
repair is known are S, Y, and Z.
2) Answer is (A)
It is possible that X only repairs radios, in which case (B) and (C) are out. If W
repairs either TVs or VCRs, but not both, (D) would not be true. U and W could
share zero or one type of machine in common, but certainly not two, so (E) is
rejected, leaving (A).
3) Answer is (D)
4) Answer is (C)
From our drawing, we look for pairs of technicians that repair exactly the same
number and types of machines. S repairs radios only, and U radios plus
something else, so (A) is impossible. Y and U both repair two types of machines,
but differ on one type at least, so (B) is out. X repairs radios but W does not, so
(D) is out. X repairs radios and Y does not, so (E) is out, leaving (C).
5) Answer is (C)
Y repairs TVs and VCRs. U repairs radios and one other type of machine. That
would have to be a TV or VCR, so Y and U would have exactly one type in
common only.
MULTILEVEL LINEAR GAMES
STRATEGY
GENERAL OBSERVATION
Multilevel linear games employ all the same deductive techniques as simple linear with
one huge exception: there is always the need to coordinate the sequence at one level
with the sequence at the other level (assuming a two level game). If this were not the
case, you would simply have two simple linear games going on simultaneously.
RECOGNITION KEYS
As with simple linear games, the key words in the game statement have to deal with
order or relative positions in some order scheme. It will usually be clear that there
are two or more (usually two) sequences to construct. At least one rule will tie the two
sequences together by introducing either required or forbidden configurations. Don’t
mistake a combo game, which has a grouping theme as well as a sequencing theme, for
a pure multilevel linear game. A point of confusion might be the rules that require the
sequences to be coordinated. They may superficially resemble some sort of selection
or classification condition.
GAME FEATURES
After you have synchronized the multiple sequences, one on top of the other, you will
want to examine the rules for secondary deductions that create “cross-blocks”. These
are partial bits of two or more sequences where either two variables must appear at
the same point in each sequence, or must not. Very frequently, the test makers impose
some other conditions on the individual variables in a cross-block so that it begins to
resemble an odd-shaped puzzle piece. The practice test diagrams will clarify this idea.
Cross-blocks are the main feature of multilevel games that distinguishes them from
ordinary linear games. While they impose a complication on determining valid
sequences, they also act to reduce the number of possible configurations, and this is
always a welcome benefit.
Multilevel Linear Game 1
Scenario
Rules
Questions
(A) Monday
(B) Tuesday
(C) Wednesday
(D) Friday
(E) Saturday
(A) There is one day on which the doctor treats patients both in the morning and in
the afternoon.
(B) The doctor conducts research on one of the days on which she lectures.
(C) The doctor conducts research on one of the days on which she treats patients.
(D) The doctor lectures on one of the days on which she treats patients.
(E) The doctor lectures on one of the days on which she operates.
4) If Doctor Yamata operates on Tuesday, then her schedule for treating patients
could be
5) Which one of the following is a pair of days on both of which Doctor Yamata must
treat patients?
1) Answer is (B)
Since she lectures on two consecutive calendar days, those days must either be
Mon/Tues, Tues/Weds, or Fri/Sat, but since she does not lecture on Saturday it
must be one of the first two options – both of which include Tuesday. * This is a
new global rule. It also follows that since the consecutive lecture must be on
either Monday or Wednesday, she must treat patients in the afternoon on both
Friday and Saturday.
2) Answer is (C)
“Could be true” – only one of these could be true, the rest are false.
We can eliminate (A) since Doctor Yamata does not operate in the afternoons.
We can eliminate (B) since she does not lecture in the mornings, and since she
does not conduct research in the afternoons we can eliminate (D). We can
eliminate (E) since we know that she operates on Wednesday mornings.
3) Answer is (E)
We know that she operates on Wednesday morning and then on either Monday
morning or Tuesday morning. We also know that she lectures on Tuesday
afternoon and on either Monday afternoon or Wednesday afternoon. Suppose
that she treats patients on Wednesday afternoon, then this means that she
lectures on Monday afternoon and Tuesday afternoon, whichever one it is will
coincide with a morning on which she operates.
4) Answer is (E)
We can rule out answer choices (A) and (C) right away since we know that she
treats patients on both Friday afternoon and Saturday afternoon. We can also
eliminate (D) since she does not treat patients on Wednesday mornings. (B) is
also incorrect since she only treats patients on one of the mornings.
5) Answer is (E)
Scenario
Eight physics students - four majors: Frank, Gwen, Henry, and Joan; and four
nonmajors: Victor, Wanda, Xavier, and Yvette - are being assigned to four laboratory
benches, numbered 1 through 4. Each student is assigned to exactly one bench, and
exactly two students are assigned to each bench. Assignments of students to benches
must conform to the following conditions:
Rules
Questions
2) If Victor is assigned to bench 2 and Wanda is assigned to bench 4, which one of the
following must be true?
3) If Gwen and Henry are not assigned to consecutively numbered benches, which one
of the following must be true?
5) If Gwen is assigned to bench 4 and Xavier is assigned to bench 3, then any one of
the following could be true EXCEPT:
7) Which one of the following could be the assignments for bench 2 and bench 4?
1) Answer is (C)
We can eliminate (B) and (D) because of the F/v/J triple. We can eliminate (E)
because G cannot be with w. Finally, we eliminate (A) since x and y are on the
same bench, yet there must be exactly one major assigned to each bench.
2) Answer is (B)
Because of the F/v/J triple we know that F and v must be on bench 2, which
means that J must be on bench 3. Since w must be on bench 4, and G cannot
be with w, G must be on bench 1.
3) Answer is (A)
Since the F/v/J triple must fill bench 2 and part of bench 3 (to keep G and H
separate), we know that v must be seated at bench 2.
4) Answer is (D)
This additional rule creates a block of four: F/v/J/w (since G cannot sit with w)
This block of four either takes up benches 2 and 3, or benches 3 and 4. So we
can rule out (B) as J has to sit at a higher numbered bench than F, so J cannot
sit at bench 2. We also rule out (C) because either F or G must sit at bench 2,
and F sits with v, while G cannot sit with w, so either way w cannot sit at bench 2.
We eliminate choice (E) because either F and v sit at bench 3, or J and 2 do.
Finally we can eliminate (A) since x must be sat with G and (E) told us that x
cannot sit at bench 3, therefore neither can G.
5) Answer is (E)
This set-up forces G to sit at bench 4 with y, which means (E) must be false.
6) Answer is (A)
If w must be seated at a lower numbered bench than J, then the F/v/J triple can
either be at benches 2 and 3, or at benches 3 and 4 (since there needs to be a
lower numbered bench left for w to sit at.) (See diagram). In the first option,
obviously H is at a lower numbered bench than F, and in option 2, we see that w
must be seated at bench 1 – with H, since G cannot sit with w. So again in this
option, H must be seated at a lower numbered bench than F.
7) Answer is (D)
Answer choice (A), splits up the F/v/J triple which must take up consecutive
benches. (B) and (E) must be false, since they both force G with w. We can
eliminate (C) since J does not sit with v. (F does).
Multilevel Linear Game 3
Scenario
During three days - Monday through Wednesday - a health officer will inspect exactly six
buildings - three hotels: Grace, Jacaranda, and Lido; and three restaurants: Seville,
Vesuvio, and Zeno. Each day, exactly two buildings are inspected: one in the morning
and one in the afternoon. Inspections must occur according to the following conditions:
Rules
Questions
1) Which of the following could be the order in which the buildings are inspected, listed
in order from Monday morning through Wednesday afternoon?
2) Which of the following could be the buildings inspected in the mornings, listed in
order from Monday through Wednesday?
3) Which one of the following is a pair of buildings that, if inspected on the same day as
each other, must be inspected on Monday?
5) If Seville is inspected on Monday morning, which one of the following must be true?
1) Answer is (D)
We can eliminate (A) since G and s cannot be inspected on the same day. (B) is
false since hotels cannot be inspected on Wednesday. (C) has J before G, and
(E) has z in the morning, but L in the afternoon.
2) Answer is (C)
We can rule out (A) since if z is in the morning, then L should be in the morning
also. (B) violates the rule that G must be inspected before J. (D) and (E) both
result in G being inspected on Monday afternoon (in order to be inspected before
J), but this violates the rule that G cannot be inspected the same day as s.
3) Answer is (B)
4) Answer is (B)
5) Answer is (D)
6) Answer is (D)
z in the morning means that L must also be in the morning (Tuesday morning).
For the afternoon slots there are two possibilities (see diagram). In both
arrangement we see that L is inspected before s.
Multilevel Linear Game 4
Scenario
On the basis of an examination, nine students - Fred, Glen, Hilary, Ida, Jan, Kathy,
Laura, Mike, and Nick - are each placed in one of three classes. The three highest
scorers are placed in the level 1 class; the three lowest scorers are placed in the level 3
class. The remaining three are placed in the level 2 class. Each class has exactly three
students.
Rules
Questions
1) How many different combinations of students could form the level 1 class?
(A) one
(B) two
(C) three
(D) four
(E) six
2) Which one of the following students could be in the level 2 class but cannot be in the
level 3 class?
(A) Fred
(B) Glen
(C) Jan
(D) Kathy
(E) Nick
3) Which one of the following students could be placed in any one of the three classes?
(A) Fred
(B) Jan
(C) Kathy
(D) Laura
(E) Mike
4) The composition of each class can be completely determined if which one of the
following pairs of students is known to be in the level 2 class?
5) Which one of the following pairs of students cannot be in the same class as Fred?
1) Answer is (B)
We deduced that I and G had to be in level 1. J and K are below each of those,
but we don’t know the exact relation between J and K. Evidently either could be
the third highest scoring student, so there would be two possibilities for the level
1 class roster.
2) Answer is (C)
M, H, and N all have scores lower then J, so J could not possibly be in level 3.
3) Answer is (C)
(B) is out, since we established that J can’t be in level 3. I, G, and F are strictly
above M, H, and N, so none of M, H, or N can be in level 1. This eliminates (E).
Likewise, I, G, and K are strictly above F and L, so F and L can’t be in level 1.
This eliminates (A) and (D). (C) remains as the only possibility.
4) Answer is (C)
If F and K are in level 2, then I, G, and J are certainly in level 1, but either L or M
could be in level 2...not definite, so (A) is out. If Fred and Mike or Laura and Mike
are in level 2, there is no decision on whether J or K is in level 1, so (B) and (E)
are out. If K and L are in level 2, then again it is certain that I, G, and J are in
level 1, but there is no way to tell whether it is F or M that is the third student in
level 2. So (D) is out and we are lft with (C).
5) Answer is (E)
H, N, and F could all be in level 3, so (A) is out. I, G, and K could be the level 1
class, leaving J, F, and L as a possible level 2 class, with M, H, and N at the
bottom, so (B) is out. Nothing wrong with I, G, J at the top and F, K, and L at
level 2, so (C) is out. L could be down in level 3 with H and N, I, G, and K could
be in level 1, and that would leave J, M, and F as a possible level 2 class, so (D)
is out. We are left with (E).
Multilevel Linear Game 5
Scenario
Four lions - F, G, H, J - and two tigers - K and M - will be assigned to exactly six stalls,
one animal per stall. The stalls are arranged as follows:
First Row: 1 2 3
Second Row: 4 5 6
The only stalls that face each other are stalls 1 and 4, stalls 2 and 5, and stalls 3 and 6.
The following conditions apply:
Rules
Questions
The fact that we have been able to isolate the only two possible configurations is an
enormous help in this game.
1) Answer is (E)
From our diagram we can see immediately that the only valid choice is k in either
stall 2 or stall 4.
2) Answer is (B)
Run the choices past our valid options. The only choice that works is (B).
3) Answer is (C)
4) Answer is (E)
Since k and H are in the same row, we know that option two is the configuration
that must be in place. M and G are in the same row, so (E).
5) Answer is (C)
Since J is in stall 3, we know that option one is in play. Running the choices by
this option reveals that G can be in stall 1 and F in stall 5, so (C).
6) Answer is (B)
Scenario
Rules
Questions
(A) 1.
(B) 2
(C) 4
(D) 7
(E) 8
2) If there is a ranch house directly opposite another ranch house, which one of the
following could be true?
5) If no house faces a house of the same style, then it must be true that house
6) If the condition requiring house 6 to be a split-level house is suspended but all other
original conditions remain the same, then any of the following could be an accurate list
of the styles of houses 2,4,6 and 8, respectively, EXCEPT:
1) Answer is (D)
From our diagram, we see that house 5 must be Tudor style. Adjacent (side by
side, not across the street) houses must have different styles, so houses 3 and 7
cannot be Tudor. 3 doesn’t appear as a choice, but 7 does.
2) Answer is (B)
If two ranch houses are across from each other, that must happen only for
houses 3 and 4, so (C) is out. The TR rule then forces house 2 to be Tudor, so
(D) is out. Also, house 1 is split-level or Tudor, so (E) is out. We know house 8 is
Tudor, so (A) is out. (B) remains.
3) Answer is (A)
If house 4 is Tudor, then house 2 cannot be, so (B) is out. House 5 is Tudor, so
(C) is out. House 7 is split-level or ranch, so (D) is out. Finally, house 8 is known
to be Tudor, so (E) is out. (A) works.
4) Answer is (A)
5) Answer is (E)
Since house 3 is ranch, this local rule prevents house 4 from being ranch, so it
must be Tudor, choice (E).
6) Answer is (A)
STRATEGY
GENERAL OBSERVATIONS
Combination games have both elements of ordering and grouping in them, and
accordingly they require skill in solving both sequencing and grouping games.. Very
often they are the most difficult logic games that appear on the LSAT. There is
usually a “killer” game among the four given each exam date, and it frequently is a
combo game. This is mentioned not to discourage you, but to make you aware that
the most efficient use of time on the logic games portion of the LSAT involves
recognizing what you can confidently do and relegating the rest to informed
guesses, where the number of answer choices per question can been reduced from
the initial five.
RECOGNITION KEYS
You are looking for linear, grouping and multilevel games among the four you are
faced with on the LSAT. This will let you start with something simpler and that you
have experience with. If a game doesn’t seem to fit any of those profiles, and if it
has both sequencing and grouping aspects, it is a hybrid or combo game. If the
game you are looking at is even missing these hybrid characteristics, you have
latched onto a miscellaneous game. Fortunately these are rare on the LSAT.
GAME FEATURES
There is very little advice to be given that is specific to combo games, other than
saving them for last unless they are unnaturally simple looking. Using the
techniques for generating secondary deductions for sequencing and grouping games
is certainly a way to start, although attention must be paid to the possibility that a
grouping rule, for example, may affect a sequencing option, or a sequencing rule
may affect a grouping option. So you will want to be on the lookout for deductions
of this sort that are themselves hybrids. The practice exams will illustrate how
this works.
Combination Game 1
Scenario
Exactly five cars - Frank’s, Marquitta’s, Orlando’s, Taishah’s, and Vinquetta’s - are
washed, each exactly once. The cars are washed one at a time, with each receiving
exactly one kind of wash: regular, super, or premium.
Rules
1) The first car washed does not receive a super wash, though at least one car does.
2) Exactly one car receives a premium wash.
3) The second and third cars washed receive the same kind of wash as each other.
4) Neither Orlando’s nor Taishah’s is washed before Vinquetta’s.
5) Marquitta’s is washed before Frank’s but after Orlando’s.
6) Marquitta’s and the car washed immediately before Marquitta’s receive regular
washes.
Questions
1) Which one of the following could be an accurate list of the cars in the order in which
they are washed, matched with type of wash received?
2) If Vinquetta’s car does not receive a premium wash, which one of the following must
be true?
(A) Orlando’s and Vinquetta’s cars receive the same kind of wash as each other.
(B) Marquitta’s and Taishah’s cars receive the same kind of wash as each
other.
(C) The fourth car washed receives a premium wash.
(D) Orlando’s car is washed third.
(E) Marquitta’s car is washed fourth.
3) If the last two cars washed receive the same kind of wash as each other, then which
one of the following could be true?
5) Which one of the following is a complete and accurate list of the cars that must
receive a regular wash?
6) Suppose that in addition to the original five cars Jabrohn’s car is also washed. If all
the other conditions hold as given, which of the following CANNOT be true?
1) Answer is (B)
We can eliminate (A) since V must be the first car washed. We can also
eliminate (C) since O must be washed before M. Option (D) has no p wash, and
(E) has no s wash.
2) Answer is (A)
V must be first with r, then we know the r/r/M triple has to go 2nd and 3rd
otherwise there would be no room for both p and s. So M goes with r in third
position, which forces O to go with r in second position. Hence, V and O both go
with r.
3) Answer is (B)
We know that V must go with p and the two doubles will be r/r and s/s and in fact
since M cannot be the last car washed the r/r/M triple must be 2nd and 3rd. This
forces the 4th and 5th cars to have s washes, and also the second car washed
must be O. This set-up rules out (A), (C), (D), and (E).
4) Answer is (E)
If the second car washed receives an s wash (the only alternative since we know
it isn’t p), then the third car washed also receives an s wash. This forces the r/r/M
triple into positions 4 and 5 which cannot be, since F or T has to be washed last.
This is a new global rule: The second car washed has an ‘r’ wash. This rule
however is not actually helpful in answering the remaining questions).
5) Answer is (B)
6) Answer is (A)
Even with the introduction of J, the second and third cars washed receive the
same type of wash (which still cannot be p since there is only one p wash).
However, if J comes after O in the order, then O must be 2nd or 3rd, but then O
would not be able to have a p wash. If J comes before O in the order, then O
must either be third (in which case again, O can not have a p wash), or O must
come immediately prior to M, but we know that that car receives an r wash. Thus
it is impossible for O to have a p wash.
Combination Game 2
Scenario
The six messages on an answering machine were each left by one of Fleure, Greta,
Hildy, Liam, Pasquale, or Theodore, consistent with the following:
Rules
Questions
1) Which one of the following could be a complete and accurate list of the messages
left on the answering machine, from first to last?
2) The first and last messages on the answering machine could be the first and second
messages left by which one of the following?
(A) Fleure
(B) Hildy
(C) Liam
(D) Pasquale
(E) Theodore
3) If Greta left the fifth message, then which one of the following messages CANNOT
have been left by Theodore?
5) If the only message Pasquale left is the fifth message, then which one of the
following could be true?
1) Answer is (D)
We can eliminate (A) because since F left a message, all of P’s messages
should be before T’s message. We eliminate (B) since G left a message and yet
F did not. (C) is false since when H is first, P must be last. (E) can be eliminated
since both H and L have left more than one message.
.
2) Answer is (A)
B) must be false, since H first means that P must be last. (C) is invalid since we
know P left a message, and so all of H’s messages must come before L’s
message. To eliminate (D) and (E), we need to notice that the question states
that the last message left was some person’s second message. (ie – not their
third). Looking at (D) we see that if P is first and last, then T left a message
before P’s second message and so F must not have left a message. It follows
that G didn’t leave a message either and we are down to just 4 people: H, L, P,
and T, but to have six messages from 4 people, the person with multiple
messages must have left 3 messages. This is why it was important to notice that
P left his second message last (and didn’t leave a third). (E) is similar. If T left his
first message first and his second message last, then there is a message from P
after T’s first message. This means there is no message from F and also no
message from G which again leaves us with only 4 people, none of whom left 3
messages so the math doesn’t add up to six.
3) Answer is (A)
G in 5th place means that all of P’s messages should be before any of T’s
messages, and so T cannot have left the first message.
4) Answer is (D)
It is possible that each person left exactly one message, and so no one left two
messages. Thus (D) need not be true.
5) Answer is (C)
(A) is false since if P didn’t leave the last message, then H didn’t leave the first
message. (B) can be eliminated since if T leaves two messages, then at least
one of these was left before P left the 5th message, which means that F didn’t
leave a message and neither did G. Once again we are down to 4 people – one
of whom must have left three messages. (D) is false since L cannot have left the
second message as it would force H to have left the first message, but we
already know (from (A)) that H didn’t leave the first message. (E) can be ruled
out since T would have to have left the 6th message, but now H and L must have
left the first two messages (H the second and L the first, since we know H didn’t
leave the first one), but if L is before H then P didn’t leave a message which
leads to a contradiction.
Combination Game 3
Scenario
A child eating alphabet soup notices that the only letters left in her bowl are one each of
these six letters: T, U, W, X, Y, and Z. She plays a game with the remaining letters,
eating them in the next three spoonfuls in accord with certain rules. Each of the six
letters must be in exactly one of the next three spoonfuls, and each of the spoonfuls
must have at least one and at most three of the letters. In addition, she obeys the
following restrictions:
Rules
Questions
1) Which of the following could be an accurate list of the spoonfuls and the letters in
each of them?
(A) first: Y
second: T, W
third: U, X, Z
(B) first: T, W
second: U, X, Y
third: Z
(C) first: T
second: U, Z
third: W, X, Z
(D) first: T, U, Z
second: W
third: X, Y
(E) first: W
second: T, X, Z
third: U, Y
2) If Y is the only letter in one of the spoonfuls, then which one of the following could be
true?
3) If the Z is in the first spoonful, then which of the following must be true?
4) Which one of the following is a complete list of letters, any one of which could be the
only letter in the first spoonful?
(A) T
(B) T, W
(C) T, X
(D) T, W, Z
(E) T, X, W, Z
5) If the T is in the second spoonful, then which one of the following could be true?
1) Answer is (B)
Checking the rules, T and U cannot be in the same spoonful, so (D) is out. U
cannot be later than X, so (E) is out. Y must be later than W, so (A) is out. W and
Y cannot be in the same spoonful so (C) is out, leaving (B).
.
2) Answer is (D)
Y can never be in the first spoonful, since W is ahead of it, so (A) is out. Since Y
is alone, Z must always be with U. But U can never be in the first spoonful
because T must precede it, so (B) is out. Looking at available configurations, T
always appears in the first spoonful, so (C) is out. W can never be in the third
spoonful, because it precedes Y. (D) is the only viable choice.
3) Answer is (E)
If Z is in the first spoonful, then, since U must go with either Z or Y and U must
go after T, it follows that U cannot be in the first spoonful. Also, we now know
that U and Y are in the same spoonful. There are two possible cases: (I) spoon
1: Z, W, and T; spoon 2: U, Y; spoon 3: X or (ii) spoon 1: Z; spoon 3: U, Y, and
X, and T or W in spoon 1 or spoon 2. The common thread is that X is in spoon 3,
which is (E).
4) Answer is (D)
What is asked for is a list of letters that could be alone in the first spoonful. From
the previous question, we know Z is certainly one of them. His eliminates (A),
(B), and (C).
5) Answer is (A)
Scenario
Rules
1) Either three or four of the seven bills must be paid on Wednesday, the rest on
Thursday.
2) Bill 1 cannot be paid on the same day as bill 5.
3) Bill 2 must be paid on Thursday.
4) Bill 4 must be paid on the same day as bill 7.
5) If bill 6 is paid on Wednesday, bill 7 must be paid on Thursday.
Questions
1) If exactly four bills are paid on Wednesday, then those four bills could be
(A) 1, 3, 4, and 6
(B) 1, 3, 5, and 6
(C) 2, 4, 5, and 7
(D) 3, 4, 5,and 7
(E) 3, 4, 6, and 7
2) Which one of the following is a complete and accurate list of the bills any one of
which could be among the bills paid on Wednesday?
(A) 3, 5, and 6
(B) 1, 3, 4, 6, and 7
(C) 1, 3, 4, 5, 6, and 7
(D) 2, 3, 4, 5, 6, and 7
(E) 1, 2, 3, 4, 5, 6, and 7
3) If bill 2 and bill 6 are paid on different days from each other, which one of the
following must be true?
(A) 1
(B) 3
(C) 4
(D) 5
(E) 7
5) If bill 4 is paid on Thursday, which one of the following is a pair of bills that could also
be paid on Thursday?
(A) 1 and 5
(B) 1 and 7
(C) 3 and 5
(D) 3 and 6
(E) 6 and 7
1) Answer is (D)
Checking the rules, bill 1 and bill 5 cannot be paid on the same day, so (B) and
(E) are out. (C) is out because bill 2 must be paid on Thursday. (A) is out
because bill 4 and bill 7 are to paid on the same day. This leaves (D).
.
2) Answer is (C)
Bill 2 has to paid on Thursday, but (D) and (E) both have it being paid on
Wednesday. From the preceding question, bills 4 and 7 could have been among
those paid on Wednesday, but in the current question, (A) rules this out, so (A)
has to go. Likewise, choice (B) doesn’t allow for bill 5 to be paid on Wednesday,
so it must be incorrect. By elimination, (C) is left.
3) Answer is (A)
Bill 2 must be paid on Thursday, so by the new local rule, bill 6 must be paid on
Wednesday. That forces bill 7 to be paid on Thursday and in turn that implies bill
4 is also paid on Thursday. We are left with bills 1 and 5, which cannot be paid
on the same day, so it is either 1 on Wednesday and 5 on Thursday or vice
versa. In either case, we would have exactly three bills paid on Wednesday,
which is choice (A).
4) Answer is (B)
The logic for the preceding question shows that bill 3 has to be paid on
Wednesday. Notice that while the local rule in question 3 is not repeated
verbatim in this question, the action of the local rule for question 4 still has the
same effect in distributing bills between the two days.
5) Answer is (B)
Since bill 4 is paid on Thursday according to the local rule, then bill 7 is also paid
on Thursday. Bill 2 is always paid on Thursday. One or the other of bills 1 and 5
must be paid on Thursday. This makes four bills paid on Thursday, so three are
paid on Wednesday. Bill 6 can’t be paid Thursday - too many bills - so (D) and
(E) are out. Bill 3 can’t be paid on Thursday for the same reason, so (C) is out.
(A) is out immediately by the global rule that they can’t be on the same day, so
(B) is correct.
6) Answer is (C )
We can recycle the argument in the last three questions one more time. Bill 4 on
Thursday forces bill 3 on Wednesday.
Miscellaneous Game 1
Scenario
Each nonstop flight offered by Zephyr Airlines departs from one and arrives at another of
five cities: Honolulu, Montreal, Philadelphia, Toronto, and Vancouver. Any two cities are
said to be connected with each other if Zephyr offers nonstop flights between them.
Each city is connected with at least one other city. The following conditions govern
Zephyr’s nonstop flights:
Rules
Questions
1) Which one of the following could be a complete and accurate list of Zephyr Airlines’
connected cities?
(A) Honolulu and Vancouver; Montreal and Toronto; Philadelphia and Vancouver;
Toronto and Vancouver
(B) Honolulu and Vancouver; Montreal and Philadelphia; Montreal and Toronto;
Philadelphia and Toronto; Toronto and Vancouver
(C) Honolulu and Philadelphia; Montreal and Toronto; Philadelphia and Toronto;
Philadelphia and Vancouver; Toronto and Vancouver
(D) Honolulu and Philadelphia; Montreal and Toronto; Philadelphia and Toronto;
Philadelphia and Vancouver; Toronto and Vancouver
(E) Honolulu and Philadelphia; Honolulu and Toronto; Montreal and Philadelphia;
Philadelphia and Vancouver
2) If exactly three cities are each connected with Philadelphia, then which one of the
following could be a pair of connected cities?
(A) Montreal and Philadelphia are connected with each other, but neither is
connected with any other city.
(B) Montreal and Toronto are connected with each other, but neither is
connected with any other city.
(C) Philadelphia and Toronto are connected with each other, but neither is
connected with any other city.
(D) Philadelphia and Vancouver are connected with each other, but neither is
connected with any other city.
(E) Toronto and Vancouver are connected with each other, but neither is
connected with any other city.
5) If Toronto is the only city that is connected with Philadelphia, then which one of the
following could be true?
(A) four
(B) five
(C) six
(D) seven
(E) eight
7) If four of the cities are each connected to the remaining city, then the cities in which
one of the following pairs must be connected with each other?
1) Answer is (A)
We can eliminate (B) since M can only be connected to one other city. (C) is invalid
since H and M connected means that T and M should also be connected. We can rule
out (D) since if P and T are connected, then P and V are not. Finally, (E) is false since
H and T cannot be connected.
.
2) Answer is (B)
Since P can only be connected to exactly one of T and V (not both), we know that P
must be connected to M, H, and only one of T and V. This means that we can rule out
(A), (C) and (D). (Note – we could also rule out (A) since H and M can never be
connected.) We can narrow down the situation further by noticing that since P is
connected to H, P must be connected to T, so P and V are not connected and we can
rule out (E).
3) Answer is (A)
We noticed this in the beginning. Since M can only be connected to one city, that city
cannot be H.
4) Answer is (A)
We can eliminate (B), (C), (D) and (E) since H would be unconnected. (At least one city
must be connected to H and that city must also be connected to T). However, it is easier
to construct a valid diagram for option (A) to show that (A) could be true. (See diagram).
5) Answer is (D)
V must be connected to H since neither T nor M can be and we are told P is only
connected to T. It follows that V is connected to T. (Any city connected to H must also
be connected to T.) We know from the beginning that H and T are not connected, so T
is connected to at least two cities, and M can be connected to either T or V without
violating any rules. Since this is a “could be true” problem, we see that (D) could be true.
6) Answer is (B)
Connections that we can never have: H-T, H-M, Both of: P-T and P-V. We have three
options: Connect P & T (and not P & V). It follows that H can be connected to both P
and V (and so V must also be connected to T). M also has one connection making a
total of five connections. Connect P & V (and not P and T). V must be connected to both
H and T in order for those two cities to be connected to somewhere else. We know P
and H cannot be connected since anywhere connected to H must also be connected to
T, and P and T are not connected in this situation. M also has one connection making a
total of four connections. Connect neither P& T nor P & V. This means that P cannot
be connected to H either. H could be connected to V, and V to T, and M to P. This
leaves a total of 3 pairs.
7) Answer is (C)
We know the city connected to four of the others cannot be M (since M is only
connected to one other), neither H nor T (since H is not connected to T), nor P (since P
cannot be connected to both T and V). This leaves V which must be connected to all of
the remaining four cities, in particular H.
Miscellaneous Game 2
Scenario
The population of a small country is organized into five clans - N, O, P, S, and T. Each
year exactly three of the five clans participate in the annual harvest ceremonies. The
rules specifying the order of participation of the clans in the ceremonies are as follows:
Rules
1) Each clan must participate at least once in any two consecutive years.
2) No clan participates for three consecutive years.
3) Participation takes place in cycles, with each cycle ending when eqach of the five
clans has participated three times. Only then does a new cycle begin.
4) No clan participates more than three times withing any cycle.
Questions
1) If the clans participating in the first year of a given cycle are N, O, and P, which one of
the following could be the clans participating in the second year of that cycle?
(A) N, O, S
(B) N, O, T
(C) N, P, S
(D) O, P, T
(E) O, S, T
2) Which one of the following can be true about the clans’ participation in the
ceremonies?
3) Any cycle for the clans participation in the ceremonies must be completed at the end
of exactly how many years?
(A) five
(B) six
(C) seven
(D) eight
(E) nine
4) Which one of the following must be true about the three clans that participate in the
ceremonies the first year?
5) If, in a particular cycle, N, O, and S participate in the ceremonies in the first year,
which one of the following must be true?
6) If, in a particular cycle, N, O, and T participate in the first year and if O and P
participate in the fourth year, any of the following could be a clan that participates in the
third year except:?
(A) N
(B) O
(C) P
(D) S
(E) T
7) If, in a particular cycle, N, O, and S participate in the ceremonies in the first year and
O, S, and T participate in the third year, then which one of the following could be the
clans that participate in the fifth year?
(A) N, O, P
(B) N, O, S
(C) N, P, S
(D) O, P, S
(E) P, S, T
Miscellaneous Game 2 - Analysis
1) Answer is (E)
S and T must participate in year 2, since they did not participate in year 1.
.
2) Answer is (C)
(A) and (B) are false since a clan cannot participate for 3 consecutive years. We
can eliminate (D) since if N and S participate in years 1, 3, and 5, then we know
that O, P, and T must participate in the in-between years (years 2 and 4).
However, one of either O, P, or T must join N and S in years 1, 3, and 5, but this
forces one of them to participate in three consecutive years (years 2, 3, and 4).
To eliminate (E) we see that if S and T participate in years 2, 3, and 5, then
neither can participate in years 1 or 4 (because no clan can participate three
years in a row, nor more than 3 times per cycle). This means that N, O, and P
must participate in years 1 and 4. The problem arises when no clan can take off
for more than one year. (We are told that each clan must participate at least
once during every two consecutive years). But we see that one of N, O, or P will
not participate again until year 4 since there is only one slot left in years 2 and 3.
3) Answer is (A)
We derived this rule based on the original rule where each clan participates
exactly 3 times per cycle.
4) Answer is (A)
(A) must be true since one of the three that participate in year 1, must also
participate in year 2 with the other two clans that didn’t participate in year 1. This
clan cannot participate in year 3 as well, since no clan can participate for three
consecutive years. This leaves at most two of the original three that participated
in year 1, to participate together in year 3.
But we could also have ruled out (B) and (E) right away: (B) is false, since only
one of them will also participate in year 2 because the two that don’t participate
in year 1 must participate in year 2. (E) is false since one of them must
participate in the third year since there is only a total of 5 and at least 3 must
participate every year.
5) Answer is (D)
Since we know that P and T must participate in the second year, they must also
participate two more times each within the five year cycle. They will participate
once again on either year 3 or year 4 (but not both since they cannot participate
three years in a row), and their third participation will be in year 5.
6) Answer is (C )
P and S must participate in year 2 (since they don’t participate in year 1), and we
are told that O and P participate in year 4, this means that P cannot also
participate in year 3 or else this would violate the 3 in a row rule.
7) Answer is (E)
We know that N, P and T must participate in the second year. (P and T since
they didn’t participate in the first year, and N since both O and S are participating
in years 1 and 3 and can’t participate three years in a row.) This leaves one N,
one O, one T, one S and two P’s to participate in years 4 and 5 (to ensure that
each clan participates 3 times in the cycle). It follows that P must participate in
both years 4 and 5, and T must participate in year 5 (since T is already
participating in years 2 and 3 and cannot participate three years in a row). The
only answer choice that has both P and T participating in year 5 is (E).
TIPS & TACTICS
• You are not penalized for guessing, so don’t leave any row on the answer
bubble sheet unmarked. Budget thirty seconds at the end of the exam
to make sure this doesn’t happen. Bear in mind that if you are sure of
only a third of your answers and you guess on two thirds, statistically,
you should get about half right overall, and this would put you at a 150
for the LSAT if your performance on the other sections was comparable.
• Questions that seem to require that you check every one of the five
answer choices in a process of elimination are designed to waste your
time. If you have to guess, guess C, D, or E. Statistically, they are
slightly more likely to be the correct choices for questions of this
nature. You can see why.
• If you can eliminate any of the answer choices, your odds of guessing
correctly improve. If you are stuck on a question but have ruled out
some answers, put a tiny “X” below any answer bubble you know is
incorrect and move on. If you have time at the end, you can return to
either nail the answer or make an informed guess. Make sure to erase
the x’s.
• Stay aware of time, but don’t be obsessed with it. Your average response
time for any question should be about ninety seconds if you are going to
attempt every question. Don’t lock up on a question...three minutes tops,
then force yourself to move on.
• If you go into the exam with the plan of writing off what appears to be
the most difficult game, your time budget per question goes to two
minutes. Of course you would still bubble answers for the difficult
game, but they would be last second guesses.
DECIDING WHERE TO START
• Classify the four games. Almost always, the high difficulty game will be
a combination (order and grouping elements together) or miscellaneous
game. That is because there are more things to coordinate in a combo
game and usually less familiarity with miscellaneous game types. Rarely
do the test makers NOT include a straight linear game, and most often
they give you a linear and grouping game. Find them and start with them.
• Look for time or relative importance words like “first” and “last” or
“before” and “after” to indicate that a game is based on ordering or
sequencing. Sometimes the test makers try to hide the nature of a game,
and sometimes they are completely open about it. The former are usually
simpler games and the latter are usually more complex and require more
extensive rules.
• Grouping games are characterized by rules that say what can go with
what. They are conspicuously lacking in statements that require some
sort of ordering or sequencing. If you see language that refers to both
grouping and ordering, then you have a hybrid or combination game.
• Matching type games are just grouping games where each variable is
assigned to a category or group all by itself.
DIAGRAMS
• Write small and write fast. The only scratch paper you have is the
question sheet itself. The game scenario, rules, and questions always
appear together only on one page. Keep all diagrams for a specific game
on the sheet which presents that game.
• The test makers pay attention to how much diagramming space you may
need. If there is little space, it is because they don’t feel an elaborate
diagram is called for. If there is a lot of space, that is a clue that the
game may be diagram intensive.
• Try to get your base diagram with all the global rules in one easily
readable place.
• Try not to clutter your base diagram with speculative drawings that
refer to specific questions or use a local rule. A quick re-do of the
primary features of your basic drawing will do as a starting point for
these situations.
• If you need to make your own abbreviations, make sure you don’t
accidentally add or omit a letter just because there appears to be a
natural order. The test makers will introduce Al, Bob, Charlie, and Ed
expecting you to abbreviate them as A, B, C, D, and E. There is no D.
• The test makers come up with strange names from time to time, so the
best thing to do is reduce everything to single letter abbreviations at
the first opportunity.
• If you have two types of objects that you need to keep track of, use
upper case for one and lower case for the other. Try to resist writing
things out. These are just “notes to self” that have to be relevant for
ten minutes tops.
• Don’t forget to make all the obvious deductions from the basic rules as
given. At the very least, write down the contrapositives of every
conditional rule. If sequential order is involved, string together, if
appropriate, any statements of what comes before what.
THE QUESTIONS
• Don’t forget that the scenario part of the game statement may contain
an implied rule by limiting the number of objects or requiring all objects
to be used in some configuration...that sort of thing.
• Scan all the questions in a game before you begin. It may happen that
the statement of a later question provides a hint at the answer to an
earlier question. This typically happens with the second or third
question presenting an “if such and such is true”. If no local rules have
been introduced, whatever this question says is true must carry back to
the acceptance question. That may eliminate some answer choices right
out of the box. There also may be a glaringly obvious “silver bullet” type
of question where you see immediately what must be true from a single
rule.
• For “could be true” type questions, start with what must be true.
Usually, the variables that appear multiple times in the rules are the
most constrained...you have limited options where to place them or group
them. These form the backbone of what must be true. The variables
that are lightly constrained or even not at all are “floaters”. You have
flexibility in placing or grouping them, and they round out the “could be
true” configurations. Don’t be put off by the fact that there may be
several acceptable variable configurations and that you cannot narrow
them down to just one. What is important on the “could be true”
questions is that just a single one of the configurations presented as an
answer choice is valid. That may be the best you can do, but it is enough
to determine the answer.
HANDY DEDUCTIONS
• In multilevel linear games watch for a hard link or tie-in between one
sequence and another. This will be a key to useful secondary rules
(deductions) because the preceding principle will operate on both
sequence levels at once.
• Don’t forget that basic arithmetic is behind many games. For example,
if three variables have to fill five positions and each variable must
appear at least once, then you can immediately say that either one
variable will appear three times or two will appear twice.
• Watch for statements in games that are “loophole closers”. These are
words or phrases like “exactly” or “one each” or “at most two” that act
to sharpen or remove the ambiguity of the rules. These often
contribute significantly to the strength of the deductions you can make
from the basic global rules. Anything that reduces the number of
allowable variable configurations is a good thing.
PRACTICE TEST
Here are four garden variety logic games culled from past examinations. Although
they did not appear together in a single test, they are representative of a
complete logic games section for a typical LSAT. Try to simulate real world
conditions and attempt them all within a 35 minute total allotted time. The answers
follow the games, but don’t peek.
You should have a good level of confidence in dealing with these games now. No one
says you have to do the games in the order given, and no one says you have to do
the questions for an individual game in the order given. It often makes sense,
however, to do the latter, since later questions may be made easier by earlier
answers. But for the games themselves, it is typical for the test makers to plant a
particularly difficult game somewhere in the mix. Finding it and saving it for last is
something you will want to do. Look all the games over and start with the one that
your gut tells you is going to be the least trouble for you. This will conserve time
and boost your confidence in dealing with the rest. Good luck on this...and
especially good luck on your real exam.
5YIWXMSRW z -J ,IEXLIV LIPTW QSZI IEGL SJ XLI TMIGIW SJ JYVRMXYVI
XLIR [LMGL SRI SJ XLI JSPPS[MRK GSYPH FI XVYI#
*SYV TISTPI{+VEGI ,IEXLIV .SWL ERH 1EVME{[MPP LIPT
IEGL SXLIV QSZI I\EGXP] XLVII TMIGIW SJ JYVRMXYVI{E VIGPMRIV %
+VEGI LIPTW QSZI XLI VIGPMRIV
E WSJE ERH E XEFPI )EGL TMIGI SJ JYVRMXYVI [MPP FI QSZIH F] &
1EVME LIPTW QSZI XLI VIGPMRIV
I\EGXP] X[S SJ XLI TISTPI ERH IEGL TIVWSR [MPP LIPT QSZI EX '
.SWL LIPTW QSZI XLI WSJE
PIEWX SRI SJ XLI TMIGIW SJ JYVRMXYVI WYFNIGX XS XLI JSPPS[MRK (
1EVME LIPTW QSZI XLI WSJE
GSRWXVEMRXW )
+VEGI LIPTW QSZI XLI XEFPI
+VEGI LIPTW QSZI XLI WSJE MJ FYX SRP] MJ ,IEXLIV LIPTW
QSZI XLI VIGPMRIV ;LMGL SRI SJ XLI JSPPS[MRK GSYPH FI E TEMV SJ TISTPI
-J .SWL LIPTW QSZI XLI XEFPI XLIR 1EVME LIPTW QSZI XLI [LS LIPT IEGL SXLIV QSZI FSXL XLI VIGPMRIV ERH XLI
VIGPMRIV XEFPI#
2S TMIGI SJ JYVRMXYVI MW QSZIH F] +VEGI ERH .SWL %
+VEGI ERH .SWL
XSKIXLIV &
+VEGI ERH 1EVME
'
,IEXLIV ERH .SWL
;LMGL SRI SJ XLI JSPPS[MRK GSYPH FI ER EGGYVEXI (
,IEXLIV ERH 1EVME
QEXGLMRK SJ IEGL TMIGI SJ JYVRMXYVI XS XLI X[S TISTPI )
.SWL ERH 1EVME
[LS LIPT IEGL SXLIV QSZI MX#
%
VIGPMRIV +VEGI ERH 1EVME WSJE ,IEXLIV ERH -J .SWL ERH 1EVME LIPT IEGL SXLIV QSZI XLI WSJE XLIR
.SWL XEFPI +VEGI ERH ,IEXLIV [LMGL SRI SJ XLI JSPPS[MRK GSYPH FI XVYI#
&
VIGPMRIV +VEGI ERH 1EVME WSJE ,IEXLIV ERH %
,IEXLIV ERH .SWL LIPT IEGL SXLIV QSZI XLI
1EVME XEFPI +VEGI ERH .SWL VIGPMRIV
'
VIGPMRIV ,IEXLIV ERH .SWL WSJE +VEGI ERH &
,IEXLIV ERH 1EVME LIPT IEGL SXLIV QSZI XLI
,IEXLIV XEFPI .SWL ERH 1EVME VIGPMRIV
(
VIGPMRIV ,IEXLIV ERH .SWL WSJE ,IEXLIV ERH '
+VEGI ERH .SWL LIPT IEGL SXLIV QSZI XLI XEFPI
1EVME XEFPI +VEGI ERH 1EVME (
+VEGI ERH 1EVME LIPT IEGL SXLIV QSZI XLI XEFPI
)
VIGPMRIV .SWL ERH 1EVME WSJE +VEGI ERH )
,IEXLIV ERH 1EVME LIPT IEGL SXLIV QSZI XLI
,IEXLIV XEFPI +VEGI ERH 1EVME XEFPI
-J .SWL ERH 1EVME LIPT IEGL SXLIV QSZI XLI VIGPMRIV XLIR
[LMGL SRI SJ XLI JSPPS[MRK QYWX FI XVYI#
%
,IEXLIV LIPTW QSZI XLI WSJE
&
.SWL LIPTW QSZI XLI WSJE +3 32 83 8,) 2)<8 4%+)
'
1EVME LIPTW QSZI XLI WSJE
(
+VEGI LIPTW QSZI XLI XEFPI
)
,IEXLIV LIPTW QSZI XLI XEFPI
Answers to Practice Test
Game 1 B, D, E, C, E, C
Game 2 A, D, B, B, E
Game 3 E, D, C ,D, D, E, E
Game 4 E, A, B, D, E